GI part 1

Pataasin ang iyong marka sa homework at exams ngayon gamit ang Quizwiz!

4. A nurse is assessing a client in an extended care facility. The nurse should recognize which of the following findings is a manifestation of an obstruction of the large intestine due to a fecal impaction? A. The client reports one bowel movement yesterday. B. The client is having small, frequent liquid stools. C. The client is flatulent. d. The client indicates vomiting once this morning.

A. A report of a bowel movement yesterday does not indicate a mechanical obstruction of the large intestine due to a fecal impaction. B. CORRECT: small, frequent liquid stools can be passed around a fecal impaction. Other manifestations include constipation and rectal pain. C. The presence of flatus does not indicate a mechanical obstruction of the large intestine due to a fecal impaction. d. A report of a single episode of vomiting does not indicate a mechanical obstruction of the large intestine due to a fecal impaction. Frequent vomiting is a manifestation of a small‐bowel obstruction.

31. The nurse is assessing a patient with abdominal pain. The nurse, who notes that there is ecchymosis around the area of umbilicus, will document this finding as a. Cullen sign. b. Rovsing sign. c. McBurney sign. d. Grey-Turner's sign.

ANS: A Cullen sign is ecchymosis around the umbilicus. Rovsing sign occurs when palpation of the left lower quadrant causes pain in the right lower quadrant. Grey Turner's sign is bruising over the flanks. Deep tenderness at McBurney's point (halfway between the umbilicus and the right iliac crest), known as McBurney's sign, is a sign of acute appendicitis.

24. A patient who underwent a gastroduodenostomy (Billroth I) 12 hours ago reports increasing abdominal pain. The patient has no bowel sounds and 200 mL of bright red nasogastric (NG) drainage in the past hour. What is the highest priority action by the nurse? a. Monitor drainage. b. Contact the surgeon. c. Irrigate the NG tube. d. Give prescribed morphine.

ANS: B Increased pain and 200 mL of bright red NG drainage 12 hours after surgery indicate possible postoperative hemorrhage, and immediate actions such as blood transfusion or return to surgery are needed (or both). Because the NG is draining, there is no indication that irrigation is needed. Continuing to monitor the NG drainage is not an adequate response. The patient may need morphine, but this is not the highest priority action.

21. Which information will the nurse include when teaching a patient with peptic ulcer disease about the effect of ranitidine (Zantac)? a. "Ranitidine absorbs the excess gastric acid." b. "Ranitidine decreases gastric acid secretion." c. "Ranitidine constricts the blood vessels near the ulcer." d. "Ranitidine covers the ulcer with a protective material."

ANS: B Ranitidine is a histamine-2 (H2) receptor blocker that decreases the secretion of gastric acid. Ranitidine does not constrict the blood vessels, absorb the gastric acid, or cover the ulcer.

23. A patient admitted with a peptic ulcer has a nasogastric (NG) tube in place. When the patient develops sudden, severe upper abdominal pain, diaphoresis, and a firm abdomen, which action should the nurse take? a. Irrigate the NG tube. b. Check the vital signs. c. Give the ordered antacid. d. Elevate the foot of the bed.

ANS: B The patient's symptoms suggest acute perforation, and the nurse should assess for signs of hypovolemic shock. Irrigation of the NG tube, administration of antacids, or both would be contraindicated because any material in the stomach will increase the spillage into the peritoneal cavity. Elevating the foot of the bed may increase abdominal pressure anddiscomfort, as well as making it more difficult for the patient to breathe.

1. Which action will the nurse include in the plan of care for a patient who is being admitted with Clostridium difficile? a. Teach the patient about proper food storage. b. Order a diet without dairy products for the patient. c. Place the patient in a private room on contact isolation. d. Teach the patient about why antibiotics will not be used.

ANS: C Because C. difficile is highly contagious, the patient should be placed in a private room, and contact precautions should be used. There is no need to restrict dairy products for this type of diarrhea. Metronidazole (Flagyl) is frequently used to treat C. difficile infections. Improper food handling and storage do not cause C. difficile.

7. A 68-yr-old male patient with a stroke is unconscious and unresponsive to stimuli. After learning that the patient has a history of gastroesophageal reflux disease (GERD), what should the nurse plan to assess more frequently than is routine? a. Apical pulse b. Bowel sounds c. Breath sounds d. Abdominal girth

ANS: C Because GERD may cause aspiration, the unconscious patient is at risk for developing aspiration pneumonia. Bowel sounds, abdominal girth, and apical pulse will not be affected by the patient's stroke or GERD and do not require more frequent monitoring than the routine.

9. After abdominal surgery, a patient with protein calorie malnutrition is receiving parenteral nutrition (PN). Which is the best indicator that the patient is receiving adequate nutrition? a. Serum albumin level is 3.5 mg/dL. b. Fluid intake and output are balanced. c. Surgical incision is healing normally. d. Blood glucose is less than 110 mg/dL.

ANS: C Because poor wound healing is a possible complication of malnutrition for this patient, normal healing of the incision is an indicator of the effectiveness of the PN in providing adequate nutrition. Blood glucose is monitored to prevent the complications of hyperglycemia and hypoglycemia, but it does not indicate that the patient's nutrition is adequate. The intake and output will be monitored, but do not indicate that the PN is effective. The albumin level is in the low-normal range but does not reflect adequate caloric intake, which is also important for the patient.

8. How should the nurse explain esomeprazole (Nexium) to a patient with recurring heartburn? a. "It reduces gastroesophageal reflux by increasing the rate of gastric emptying." b. "It neutralizes stomach acid and provides relief of symptoms in a few minutes." c. "It coats and protects the lining of the stomach and esophagus from gastric acid." d. "It treats gastroesophageal reflux disease by decreasing stomach acid production."

ANS: D The proton pump inhibitors decrease the rate of gastric acid secretion. Promotility drugs such as metoclopramide (Reglan) increase the rate of gastric emptying. Cryoprotective medications such as sucralfate (Carafate) protect the stomach. Antacids neutralize stomach acid and work rapidly.

6. Which statement to the nurse from a patient with jaundice indicates a need for teaching? a. "I used cough syrup several times a day last week." b. "I take a baby aspirin every day to prevent strokes." c. "I take an antacid for indigestion several times a week" d. "I use acetaminophen (Tylenol) every 4 hours for pain."

ANS: D Chronic use of high doses of acetaminophen can be hepatotoxic and may have caused the patient's jaundice. The other patient statements require further assessment by the nurse but do not indicate a need for patient education.

37. Four hours after a bowel resection, a 74-yr-old male patient with a nasogastric tube to suction complains of nausea and abdominal distention. The first action by the nurse should be to a. auscultate for hypotonic bowel sounds. b. notify the patient's health care provider. c. check for tube placement and reposition it. d. remove the tube and replace it with a new one.

ANS: C Repositioning the tube will frequently facilitate drainage. Because this is a common occurrence, it is not appropriate to notify the health care provider unless other interventions do not resolve the problem. Information about the presence or absence of bowel sounds will not be helpful in improving drainage. Removing the tube and replacing it are unnecessarily traumatic to the patient, so that would only be done if the tube was completely occluded.

17. A patient has peptic ulcer disease that has been associated with Helicobacter pylori. About which medications will the nurse plan to teach the patient? a. Sucralfate (Carafate), nystatin, and bismuth (Pepto-Bismol) b. Metoclopramide (Reglan), bethanechol (Urecholine), and promethazine c. Amoxicillin (Amoxil), clarithromycin (Biaxin), and omeprazole (Prilosec) d. Famotidine (Pepcid), magnesium hydroxide (Mylanta), and pantoprazole (Protonix)

ANS: C The drugs used in triple drug therapy include a proton pump inhibitor such as omeprazole and the antibiotics amoxicillin and clarithromycin. The other combinations listed are not included in the protocol for H. pylori infection.

28) A client is diagnosed with a gastric ulcer secondary to NSAID overuse. The nurse would question which healthcare provider order? 1. Clarithromycin 2. Omeprazole 3. Famotidine 4. Ranitidine

Answer: 1 Explanation: A combination of antibiotics is concurrently used to eradicate H. pylori and help decrease antibiotic resistance. Those with peptic ulcers who are not infected with H. pylori have been shown to have a worse outcome when receiving H. pylori treatment. Page Ref: 1088

12) A 48-year-old male is returned from an endoscopic procedure with a diagnosis of a duodenal ulcer. The nurse is aware that the most likely causative agent for this ulcer is: 1. H. pylori. 2. prolonged NSAID use. 3. intermittent use of glucocorticoids. 4. chronic stress.

Answer: 1 Explanation: H. pylori—associated ulcers are most likely to be duodenal. Page Ref: 1085

26) A client with GERD must be cautioned against the overuse of antacids to avoid which syndrome? 1. Metabolic alkalosis 2. Metabolic acidosis 3. Respiratory alkalosis 4. Respiratory acidosis

Answer: 1 Explanation: Ingestion of large amounts of antacid can leave a client with a pH base excess. This contributes to metabolic alkalosis. Page Ref: 1094

21) The nurse is planning care for a client with PUD. What treatment goal should be included in the plan of care for this client? 1. Prevent recurrence of the disease. 2. Patient reports tolerating symptoms. 3. Reduce the level of H. pylori by one third. 4. Promote stasis of the ulcer.

Answer: 1 Explanation: Preventing the recurrence of the disease is a treatment goal. Page Ref: 1088

25) A client is prescribed an H2-receptor antagonist. The nurse teaches this client that the purpose of the drug is to: 1. reduce gastric acid secretion in the stomach. 2. prevent the vasodilating effects of histamine. 3. alleviate nasal allergy symptoms. 4. prevent the release of histamine from MAST cells.

Answer: 1 Explanation: The H2 receptors, located on the parietal cells in the stomach, promote acid secretion when activated. Page Ref: 1092

39) The nurse is caring for a teenager with PUD. The nurse recommends that client begin taking ranitidine (Zantac) OTC because the client is older than ________ year(s). Record your answer rounding to the nearest whole number.

Answer: 12 Explanation: OTC administration of ranitidine is contraindicated in children younger than 12 years of age. Children older than 12 years, however, may be prescribed the medication for the treatment of PUD OTC. Page Ref: 1094

11) A client new to the clinic reports having peptic ulcer disease and a weight loss of 14 pounds in 2 months. The symptom of weight loss would be most consistent with: 1. a duodenal ulcer. 2. a gastric ulcer. 3. frequent exercise. 4. anorexia nervosa.

Answer: 2 Explanation: Anorexia, weight loss, and vomiting are more common with gastric ulcers. Page Ref: 1086

36) A client is prescribed omeprazole (Prilosec). The nurse prepares to perform client teaching based on the knowledge that omeprazole is a: 1. nicotinic receptor blocker. 2. proton pump inhibitor. 3. muscarinic receptor blocker. 4. histamine-2 receptor agonist.

Answer: 2 Explanation: Omeprazole is a proton pump inhibitor. Page Ref: 1092

16) A nurse is performing discharge teaching for a client being discharged with a diagnosis of gastroesophageal reflux disease. The nurse should teach the client to eliminate what foods from the diet? 1. Mashed potatoes 2. Tomato sauce 3. Bananas 4. Toast

Answer: 2 Explanation: Substances that worsen GERD symptoms include caffeine, alcohol, citrus fruits, tomato-based products, onions, carbonated beverages, spicy foods, chocolate, and smoking. Page Ref: 1086

15) A client has been diagnosed with inflammatory bowel disease (IBD). The nurse anticipates that the healthcare provider may prescribe a drug in which category to treat this disorder? 1. Symptom-targeted therapy, antidepressants, and antianxiety drugs 2. 5-aminosalicylic acid (5-ASA) agents, immunosuppressive agents, and biologic therapies 3. Antispasmodics, serotonin agents, and biologic therapies 4. 5-aminosalicylic acid (5-ASA) agents, symptom-targeted therapy, and antianxiety agents

Answer: 2 Explanation: The pharmacotherapy for inflammatory bowel disease (IBD) includes 5-aminosalicylic acid (5-ASA) agents, immunosuppressive agents, and biologic therapies. Page Ref: 1112

6) The nurse is caring for an adult man diagnosed with a peptic ulcer. The nurse states that the most likely causative agent is: 1. Cryptococcus. 2. Pneumocystis carinii. 3. Helicobacter pylori. 4. Mycobacterium avium.

Answer: 3 Explanation: Helicobacter pylori is associated with 70% of peptic ulcers. Page Ref: 1084

41) A client with known peptic ulcer disease is placed on a clear-liquid diet pending an endoscopy. What proton pump inhibitor can be given in a liquid form to this client? 1. Famotidine (Pepcid) 2. Ranitidine (Zantac) 3. Lansoprazole (Prevacid) 4. Esomeprazole (Nexium)

Answer: 3 Explanation: Lansoprazole is available in a liquid suspension. Page Ref: 1092

The nurse determines a patient undergoing ileostomy surgery understands the procedure when the patient states a. "I should only have to change the pouch every 4 to 7 days." b. "The drainage in the pouch will look like my normal stool." c. I may not need to wear a drainage pouch if I irrigate it daily. d. Limiting my fluid intake should decrease the amount of output.

Correct answer: a Rationale: Because ileostomy drainage is a liquid to thin paste, the patient will need to wear a drainage bag at all times. The patient should use an open-ended drainable pouch. It is worn for 4 to 7 days. Output from a sigmoid colostomy resembles normally formed stool, and some patients are able to regulate emptying time so they do not need to wear an ostomy pouch.

A patient is jaundiced and her stools are clay colored (gray). This is most likely related to a. decreased bile flow into the intestine. b. increased production of urobilinogen. c. increased production of cholecystokinin. d. increased bile and bilirubin in the blood

Correct answer: a Rationale: Bile is produced by the hepatocytes and is stored and concentrated in the gallbladder. When bile is released from the common bile duct, it enters the duodenum. In the intestines, bilirubin is reduced to stercobilinogen and urobilinogen by bacterial action. Stercobilinogen accounts for the brown color of stool. Stools may be clay-colored if bile is not released from the common bile duct into the duodenum. Jaundice may result if the bilirubin level in the blood is elevated.

Which instructions would the nurse include in a teaching plan for a patient with mild gastroesophageal reflux disease (GERD)? a. "The best time to take an as-needed antacid is 1 to 3 hours after meals." b. "A glass of warm milk at bedtime will decrease your discomfort at night." c. "Do not chew gum; the excess saliva will cause you to secrete more acid." d. "Limit your intake of foods high in protein because they take longer to digest."

Correct answer: a Rationale: Patients who use an as-needed antacid should do so 1 to 3 hours after eating. Teach patients that the increased saliva production associated with chewing gum will help with GERD symptoms. The patient should not eat meals within 3 hours of bedtime. Some foods, such as red wine, decrease lower esophageal sphincter pressure and aggravate symptoms. Milk increases gastric acid secretion. There is no need for the patient to limit protein intake.

M.J. calls the clinic and tells the nurse that her 85-year-old mother has been nauseated all day and has vomited twice. Before the nurse hangs up and calls the HCP, she should tell M.J. to a. administer antiemetic drugs and observe skin turgor. b. give her mother sips of water and elevate the head of her bed to prevent aspiration. c. offer her mother a high-protein liquid supplement to drink to maintain her nutritional needs. d. offer her mother large quantities of Gatorade to decrease the risk of sodium depletion.

Correct answer: b Rationale: Excessive replacement of fluid and electrolytes may result in adverse consequences for an older person who has heart failure or renal disease. An older adult with a decreased level of consciousness may be at high risk for aspiration of vomitus. The elderly are particularly susceptible to the central nervous system (CNS) side effects of antiemetic drugs; these drugs may produce confusion. Dosages should be reduced and efficacy closely evaluated. Older patients are more likely to have cardiac or renal insufficiency, which increases their risk for life-threatening fluid and electrolyte imbalances. High-protein drinks and high-sodium liquids may be contraindicated.

A nursing intervention that is most appropriate to decrease postoperative edema and pain after an inguinal herniorrhaphy is a. applying a truss to the hernia site. b. allowing the patient to stand to void. c. supporting the incision during coughing. d. applying a scrotal support with an ice bag.

Correct answer: d Rationale: Scrotal edema is a painful complication after an inguinal hernia repair. Scrotal support with application of an ice bag may help relieve pain and edema.

When evaluating the patient's understanding about the care of the ileostomy, which statement by the patient indicates the patient needs more teaching? a. "I will be able to regulate when I have stools." b. "I will be able to wear the pouch until it leaks." c. "The drainage from my stoma can damage my skin." d. "Dried fruit and popcorn must be chewed very well."

a. "I will be able to regulate when I have stools." An ileostomy is in the ileum and drains liquid stool frequently, unlike a colostomy, which has more formed stool the farther distal the ostomy is in the colon. The ileostomy pouch is usually worn for 4 to 7 days or until it leaks. It must be changed immediately if it leaks because the drainage is very irritating to the skin. To avoid obstruction, popcorn, dried fruit, coconut, mushrooms, olives, stringy vegetables, food with skin, and meats with casings must be chewed extremely well before swallowing because of the narrow diameter of the ileostomy lumen.

Which statement by a patient with dumping syndrome should lead the nurse to determine that further dietary teaching is needed? a. "I should eat bread and jam with every meal." b. "I should avoid drinking fluids with my meals." c. "I should eat smaller meals about 6 times a day." d. "I need to lie down for 30 to 60 minutes after my meals."

a. Dietary control durnping syndrome includes small, frequent meals With low carbohydrate content and elimination of fluids with meals. The patient should also lie down for 30 to 60 minutes after meals. These measures help delay stomach emptying, preventing the rapid movement of a high-carbohydrate food bolus into the small intestine.

Patient-Centered Care: The nurse formulates the nursing diagnosis of acute pain from the effects of medication and decreased GI motility for a postoperative patient with abdominal pain and distension with an inability to pass flatus. Which nursing intervention is most appropriate for this patient? a. Ambulate the patient more frequently. b. Assess the abdomen for bowel sounds. c. Place the patient in high Fowler's position. d. Withhold opioids because they decrease bowel motility.

a. The abdominal pain and distention that occur from the decreased motility of the bowel should be treated with increased ambulation and frequent position changes to increase peristalsis. If the pain is severe, cholinergic drugs, rectal tubes, or application of heat to the abdomen may be prescribed. Assessment of bowel sounds is not an intervention to relieve the pain, and a high Fowler's position is not indicated. Opioids may still be necessary for pain control, and motility can be increased by other means.

Which statements describe the use of antacids for peptic ulcer disease (select all that apply)? a. used on patients with verified H. Pylori b. neutralize HCI in the stomach c. produce quick. Short-lived relief of heartburn d. cover the ulcer, protecting it from erosion by acids e. high incidence of side effects and contraindications f. may be given hourly after an acute phase of GI bleeding

b, c, f. Antacids provide a quick, short-lived relief of heartburn by neutralizing HCI in the stomach that prevents the conversion of pepsinogen to pepsin. Antacids may be given hourly, orally or through an NG tube, after an acute phase of GI bleeding to neutralize HCI in the stomach. Amoxicillin/clarithromycin/omeprazole are used in patients with verified H. pylori. Sucralfate (Carafate) covers the ulcer to protect it from acid erosion. Side effects are manageable .

Which esophageal disorder is described as a precancerous lesion and is associated with GERD? a. Achalasia b. Barrett's esophagus c. Esophageal strictures d. Esophageal diverticula

b. Barrett's esophagus is an esophageal metaplasia primarily related to gastroesophageal reflux disease (GERD). Achalasia is a rare chronic disorder with delayed emptying of the lower esophagus and is associated with squamous cell cancer. Esophageal strictures are narrowing of the esophagus from scarring by many causes. Esophageal diverticula are saclike outpouchings of 1 or more layers of the esophagus. They often occur above the esophageal sphincter.

An important nursing intervention for a patient with a small intestinal obstruction who has an NG tube is to? a. offer ice chips to suck as needed. b. provide mouth care frequently. c. irrigate the tube with normal saline every 8 hours. d. keep the patient supine with the head of the bed elevated 30 degrees.

b. Mouth care should be done frequently for the patient with a small intestinal obstruction who has an NG tube because of vomiting, fecal taste and odor, and mouth breathing. No ice chips are allowed when a patient is NPO because of a bowel obstruction. The NG tube should be checked for patency and irrigated only as ordered. The position of the patient should be one of comfort.

The nurse should administer an as-needed dose of magnesium hydroxide after noting what information when reviewing a patient's medical record? a. Abdominal pain and bloating b. No bowel movement for 3 days c. A decrease in appetite by 50% over 24 hours d. Muscle tremors and other signs of hypomagnesemia

b. No bowel movement for 3 days Magnesium hydroxide is an osmotic laxative that produces a soft, semisolid stool usually within 15 minutes to 3 hours. This medication would benefit the patient who has not had a bowel movement for 3 days. It would not be given for abdominal pain and bloating, decreased appetite, or signs of hypomagnesemia.

Which patient is most likely to be diagnosed with short bowel syndrome? a. History of ulcerative colitis b. Extensive resection of the ileum c. Diagnosis of irritable bowel syndrome d. d. Colectomy performed for cancer of the bowel

b. Short bowel syndrome results from extensive resection Of portions of the small bowel and would occur if a patient had an extensive resection of the ileum. The other conditions primarily affect the large intestine and result in fewer and less severe symptoms.

The nurse is assessing a patient admitted with a possible bowel obstruction. Which assessment finding would be expected in this patient? a. Tympany to abdominal percussion b. Aortic pulsation visible in epigastric region c. High-pitched sounds on abdominal auscultation d. Liver border palpable 1 cm below the right costal margin

c. High-pitched sounds on abdominal auscultation The bowel sounds are higher pitched (rushes and tinkling) when the intestines are under tension, as in intestinal obstruction. Bowel sounds may also be diminished or absent with an intestinal obstruction. Normal findings include aortic pulsations on inspection and tympany with percussion, and the liver may be palpable 1 to 2 cm along the right costal margin.

The patient has peritonitis, which is a major complication of ruptured appendix. What treatment should the nurse plan to include? a. Peritoneal lavage b. Peritoneal dialysis c. IV fluid replacement d. Increased oral fluid intake

c. IV fluid replacement along with antibiotics, NG suction, analgesics, and potential surgery would be expected. Peritoneal lavage may be used to determine abdominal trauma. Peritoneal dialysis would not be performed. Oral fluids would be avoided with peritonitis.

What laboratory findings are expected in ulcerative colitis because of diarrhea and vomiting? a. Increased albumin b. Elevated white blood cells (WBCs) c. Decreased serum Na+, K+, Mg+, CIA and HCOi d. Decreased hemoglobin (Hgb) and hematocrit (Hct)

c. In the patient with ulcerative colitis, decreased serum Na+ K+ Mg+, Cl-, and HC03- are a result of diarrhea and vomiting. Hypoalbuminemia may be present in severe disease. Elevated white blood cell (WBC) counts occur with toxic megacolon. Decreased hemoglobin (Hgb) and hematocrit (Hct) occur with bloody diarrhea, leading to iron-deficiency anemia.

A 74-yr-old female patient with osteoporosis is diagnosed with gastroesophageal reflux disease (GERD). Which over-the-counter medication to treat GERD should be used with caution? a. Sucralfate b. Cimetidine c. Omeprazole d. Metoclopramide

c. Omeprazole There is a potential link between proton pump inhibitors (PPIs) (e.g., omeprazole) use and bone metabolism. Long-term use or high doses of PPIs may increase the risk of fractures of the hip, wrist, and spine.

How should the nurse teach the patient with a hiatal hernia or GERD to control symptoms? a. Drink 10 to 12 ounces of water with each meal. b. Space 6 small meals a day between breakfast and bedtime. c. Sleep with the head of the bed elevated on 4- to 6-inch blocks. d. Perform daily exercises of toe-touching, sit-ups, and weight lifting.

c. The use of blocks to elevate the head of the bed facilitates gastric emptying by gravity and is strongly recommended to prevent nighttime reflux. Liquids should be taken between meals to prevent gastric distention with meals. Small meals should be eaten frequently, but patients should not eat at bedtime or lie down for 2 to 3 hours after eating. Activities that involve increasing intraabdominal pressure, such as bending over, lifting, or wearing tight clothing, should be avoided.

Collaboration: The registered nurse (RN) coordinating the care for a patient who is 2 days postoperative following an abdominal-perineal resection (APR) with colostomy may delegate which interventions to the licensed practical nurse (LPN) (select all that apply)? a. Irrigate the colostomy. b. Teach ostomy and skin care. c. Assess and document stoma appearance. d. Monitor and record the volume, color, and odor of the drainage. e. Empty the ostomy bag and measure and record the amount of drainage.

d, e. The LPN can monitor and record observations related to the drainage and can measure and record the amount. The LPN could also monitor the skin around the stoma for breakdown. LPNs can irrigate a colostomy in a stable patient, but this patient is only 2 days postoperative. The other actions are responsibilities of the RN (teaching, assessing stoma, and developing a care plan).

A patient reports severe pain when the nurse assesses for rebound tenderness. What may this assessment finding indicate? a. Hepatic cirrhosis b. Hypersplenomegaly c. Gallbladder distention d. Peritoneal inflammation

d. Peritoneal inflammation When palpating for rebound tenderness, the problem area of the abdomen will produce pain and severe muscle spasm when there is peritoneal inflammation. Hepatic cirrhosis, hypersplenomegaly, and gallbladder distention do not manifest with rebound tenderness.

A patient who is unable to swallow because of progressive amyotrophic lateral sclerosis is prescribed enteral nutrition through a newly placed gastrostomy tube. Which task is appropriate for the nurse to delegate to unlicensed assistive personnel (UAP)? a. Irrigate the tube between feedings. b. Provide wound care at the gastrostomy site. c. Administer prescribed liquid medications through the tube. d. Position the patient with a 45-degree head of bed elevation.

d. Position the patient with a 45-degree head of bed elevation. UAP may position the patient receiving enteral feedings with the head of bed elevated. A licensed practical nurse/licensed vocational nurse or an RN could perform the other activities.

3. What condition should the nurse anticipate when caring for a patient with a history of a total gastrectomy? a. Constipation b. Dehydration c. Elevated total serum cholesterol d. Cobalamin (vitamin B12) deficiency

ANS: D The patient with a total gastrectomy does not secrete intrinsic factor, which is needed for cobalamin (vitamin B12) absorption. Because the stomach absorbs only small amounts of water and nutrients, the patient is not at higher risk for dehydration, elevated cholesterol, or constipation.

An 85-year-old woman seen in the primary care provider's office for a well check complains of difficulty swallowing. What common effect of aging should the nurse assess for as a possible cause? Anosmia Xerostomia Hypochlorhydria Salivary gland tumor

Xerostomia (decreased saliva production), or dry mouth, affects many older adults and may be associated with difficulty swallowing (dysphagia). Anosmia is loss of sense of smell. Hypochlorhydria, a decrease in stomach acid, does not affect swallowing. Salivary gland tumors are not common.

For the patient hospitalized with inflammatory bowel disease (IBD), which treatments would be used to rest the bowel (select all that apply)? a. NPO b. IV fluids c. Bed rest d. Sedatives e. NG suction f. Parenteral nutrition

a, b, e, f. With an acute exacerbation of inflammatory bowel disease (IBD), to rest the bowel the patient will be NPO, receive IV fluids and parenteral nutrition, and have NG suction. Sedatives may be used to alleviate stress. Enteral nutrition will be used as soon as possible.

27. Which medications are used to decrease gastric or HCI secretion (select all that apply)? a. Famotidine (Pepcid) b. Sucralfate (Carafate) c. Omeprazole (Prilosec) d. Misoprostol (Cytotec) e. Bethanechol (Urecholine)

a, c, d. Famotide (Pepcid) reduces HC1 secretion by blocking histamine and omeprazole (Prilosec) decreases gastric acid secretion by blocking adenosine triphosphatase (ATPase) enzyme. Misoprostol (Cytotec) has antisecretory effects. Sucralfate (Carafate) coats the ulcer to protect it from acid erosion. Bethanechol (Urecholine) for GERD increases LES pressure and facilities gastric empty

The nurse is planning to teach the patient with gastroesophageal reflux disease (GERD) about foods or beverages that decrease LES pressure. What should be included in this list (select all that apply)? a. Alcohol b. Root beer c. Chocolate d. Citrus fruits e. fatty foods f. Cola sodas

a, c, e, f. Alcohol, chocolate, fatty foods, and cola sodas (caffeine) as well as peppermint and spearmint will decrease lower esophageal sphincter (LES) pressure. Root beer and herbal tea do not have caffeine. Citrus fruits will not affect LES pressure.

A 20-year-old patient with a history of Crohn's disease comes to the clinic with persistent diarrhea. What are common characteristics of Crohn's disease (selæt all that apply)? a. Weight loss b. Rectal bleeding c. Abdominal pain d. Toxic megacolon e. Has segmented distribution f. Involves the entire thickness of the bowel wall

a, c, e, f. Crohn's disease may have severe weight loss, crampy abdominal pain, and segmented distribution through the entire wall of the bowel. Rectal bleeding and toxic megacolon are more often seen with ulcerative colitis.

1. Vasopressin 0.2 unit/min infusion is prescribed for a patient with acute arterial gastrointestinal (GI) bleeding. The vasopressin label states vasopressin 100 units/250 mL normal saline. How many mL/hr will the nurse infuse?

ANS: 30 There are 0.4 unit/1 mL. An infusion of 30 mL/hr will result in the patient receiving 0.2 units/min as prescribed.

1 The patient has developed severe diarrhea following 4 days of self- administered antacid preparation. The nurse suspects that the diarrhea may be caused by which type of antacid? A. Aluminum compounds B. Magnesium compounds C. Calcium compounds D. Sodium compounds

1 Answer: 2 Rationale: Magnesium compounds, especially in higher doses, often cause diarrhea. Options 1, 3, and 4 are incorrect. Aluminum compounds and calcium compounds may cause constipation. Sodium compounds may cause flatulence. Cognitive Level: Applying; Client Need: Physiological Integrity; Nursing Process: Evaluation

1 The patient is taking diphenoxylate with atropine (Lomotil). What does the nurse assess when monitoring for therapeutic effects? A. Reduction of abdominal cramping B. Minimal passage of flatus C. Decrease in loose, watery stools D. Increased bowel sounds

1 Answer: 3 Rationale: Diphenoxylate with atropine is given for diarrhea. The patient should report a decrease in the number of loose, watery stools after administration. Options 1, 2, and 4 are incorrect. Although diphenoxylate with atropine may decrease abdominal cramping and gas as a result of slowed peristalsis, it is not the main therapeutic effect desired from this drug. Slowing peristalsis may cause a decrease in bowel sounds rather than an increase. Cognitive Level: Analyzing; Client Need: Physiological Integrity; Nursing Process: Evaluation

1. A nurse is caring for a client following a paracentesis. Which of the following findings indicate the bowel was perforated during the procedure? A. Client report of upper chest pain B. decreased urine output C. Pallor d. Temperature elevation

1. A. A report of sharp, constant abdominal pain is associated with bowel perforation. B. decreased urine output is associated with bladder perforation during a paracentesis. C. Pallor may indicate hypovolemia related to fluid removal of ascites fluid during the procedure. d. CORRECT: Fever is an indication of bowel perforation during a paracentesis.

1. A nurse in the emergency department is completing an assessment of a client who has suspected stomach perforation due to a peptic ulcer. Which of the following findings should the nurse expect? (select all that apply.) A. rigid abdomen B. Tachycardia C. elevated blood pressure d. Circumoral cyanosis e. reboundtenderness

1. A. CORRECT: Manifestations of perforation include a rigid, board‐like abdomen. B. CORRECT: Tachycardia occurs due to gastrointestinal bleeding that accompanies a perforation. C. Hypotension is an expected finding in a client who has a perforation and bleeding. d. Circumoral cyanosis is not a manifestation of perforation. e. CORRECT: rebound tenderness is an expected finding in a client who has a perforation.

14. When considering enteral nutrition (EN) for a patient with severe protein-calorie malnutrition, what is an advantage of a gastrostomy tube versus a nasogastric (NG) tube? a. There is less irritation to the nasal and esophageal mucosa. b. The patient experiences the sights and smells associated with eating. c. Aspiration resulting from reflux of formulas into the esophagus is less common. d. Routine checking for placement is not needed because gastrostomy tubes do not become displaced.

14. a. Standard nasogastric (NG) tubes are only used for enteral nutrition (EN) for short-term feeding problems because prolonged therapy can result in irritation and erosion of the mucosa of the upper GI tract. Gastric reflux and the potential for aspiration can occur with both NG and gastrostomy feeding tubes. Both tubes deprive the patient of the sensations associated with eating and can become displaced.

18. What is an indication for parenteral nutrition that is not an appropriate indication for enteral tube feedings? a. Head and neck cancer b. Hypermetabolic states c. Malabsorption syndrome d. Protein-calorie malnutrition

18. c. In malabsorption syndromes, foods that are ingested into the intestinal tract cannot be digested or absorbed and tube feedings infused into the intestinal tract would not be absorbed. All of the other conditions can be treated with enteral or parenteral nutrition, depending on the patient's need

2 Omeprazole (Prilosec) is prescribed for a patient with gastroesophageal reflux disease. The nurse would monitor a reduction in which symptom to determine if the drug therapy is effective? Select all that apply. A. Dysphagia B. Dyspepsia C. Appetite D. Nausea E. Belching

2 Answer: 1, 2, 4, 5 Rationale: Symptoms of GERD include dysphagia, dyspepsia, nausea, belching, heartburn, and chest pain. Option 3 is incorrect. The nurse would not expect a decrease in the patient's appetite due to this medication. Cognitive Level: Applying; Client Need: Physiological Integrity; Nursing Process: Evaluation

2 The patient who is taking sulfasalazine (Azulfidine) develops a sore throat, bruising, and severe fatigue. The nurse determines that the patient is most likely experiencing drug-induced: A. Stevens-Johnson syndrome. B. Blood dyscrasias. C. Idiosyncratic reaction. D. Hypersensitivity response.

2 Answer: 2 Rationale: One adverse effect of sulfasalazine is blood dyscrasias, which may include anemia, leukopenia, and thrombocytopenia. Fever, an increase in bruising, and sore throat are all possible symptoms of these decreased cell counts. Options 1, 3, and 4 are incorrect. Stevens-Johnson syndrome results in inflammation of the skin and mucous membranes and includes a sunburn-like appearance, blisters, and possible exfoliation of the dermis. Idiosyncratic reactions are aberrant reactions that cannot be explained by the known pharmacologic action of the drug and occur only in a small percentage of the population. This patient's symptoms are well documented as adverse effects. Hypersensitivity responses are due to stimulation of the immune system and are invoked by an antigen or antibody response. The symptoms presented do not reflect hypersensitivity to the drug. Cognitive Level: Analyzing; Client Need: Physiological Integrity; Nursing Process: Evaluation

4. A nurse is completing discharge teaching with a client who is 3 days postoperative following a transverse colostomy. Which of the following should the nurse include in the teaching? A. Mucus will be present in stool for 5 to 7 days after surgery. B. expect 500 to 1,000 mL of semi‐ liquid stool after 2 weeks. C. stoma should be moist and pink. d. Change the ostomy bag when it is 3⁄4 full.

4. A. Mucus and blood can be present for 2 to 3 days after surgery. B. Output should become stool‐like, semi‐formed, or formed within days to weeks. C. CORRECT: A pink, moist stoma is an expected finding for a colostomy. d. The ostomy bag should be changed when it is 1⁄4 to 1⁄2 full.

2. A nurse is planning care for a client who has a new prescription for total parenteral nutrition (TPN). Which of the following interventions should be included in the plan of care? (select all that apply.) A. Obtain a capillary blood glucose four times daily. B. Administer prescribed medications through a secondary port on the TPN iV tubing. C. Monitor vital signs three times during the 12‐hr shift. d. Change the TPN iV tubing every 24 hr. e. ensure a daily aPTT is obtained.

2. A. CORRECT: The client is at risk for hyperglycemia during the administration of TPN and can require supplemental insulin. B. No other medications or fluids should be administered through the iV tubing being used to administer TPN due to the increased risk of infection and disruption of the rate of TPN infusion. C. CORRECT: Vital signs are recommended every 4 to 8 hr to assess for fluid volume excess and infection. d. CORRECT: it is recommended to change the iV tubing that is used to administer TPN every 24 hr. e. aPTT measures the coagulability of the blood, which is unnecessary during the administration of TPN.

20. Priority Decision: The nurse is caring for a patient receiving 1000 mL of parenteral nutrition solution over 24 hours. When it is time to change the solution, 150 mL remain in the bottle. What is the most appropriate action by the nurse? a. Hang the new solution and discard the unused solution. b. Open the IV line and rapidly infuse the remaining solution. c. Notify the health care provider for instructions regarding the infusion rate. d. Wait to change the solution until the remaining solution infuses at the prescribed rate.

20. a. Remaining solution should be discarded. Bacterial growth occurs at room temperature in nutritional solutions. Therefore solutions must not be infused for longer than 24 hours. Speeding up the solution may cause hyperglycemia and should not be done. The HCP does not need to be notified as the rate is determined to meet the patient's nutritional needs.

3 The nurse is scheduling the patient's daily medication. When would be the most appropriate time for the patient to receive proton pump inhibitors? A. At night B. After fasting at least 2 hours C. About 1/2 hour before a meal D. About 2 to 3 hours after eating

3 Answer: 3 Rationale: The proton pump is activated by food intake. Thus, administering it about 20 to 30 minutes before the first major meal of the day allows peak serum levels to coincide with when the maximum levels of pumps are activated, allowing maximum efficiency of the PPI. Options 1, 2, and 4 are incorrect. The proton pumps are less active at night, in the fasting state, or between meals. Cognitive Level: Applying; Client Need: Physiological Integrity; Nursing Process: Planning

3 Ondansetron (Zofran) has been ordered prior to chemotherapy for a patient receiving treatment for lymphoma. Prior to administering this drug, the nurse will review the patient's past medical history for what condition? A. Allergy to soy or soy products B. History of chronic constipation C. Glaucoma D. Cardiac dysrhythmias

3 Answer: 4 Rationale: Ondansetron is known to prolong the QT interval and may cause cardiac dysrhythmias. Options 1, 2, and 3 are incorrect. An allergy to soy or soy products, chronic constipation, or glaucoma does not present contraindications to the drugs. Cognitive Level: Analyzing; Client Need: Physiological Integrity; Nursing Process: Evaluation

3. A nurse is providing care to a client who is 1 day postoperative following a paracentesis. The nurse observes clear, pale‐yellow fluid leaking from the operative site. Which of the following is an appropriate nursing intervention? A. Place a clean towel near the drainage site. B. Apply a dry, sterile dressing. C. Apply direct pressure to the site. d. Place the client in a supine position.

3. A. Cover the operative site to prevent infection and allow for assessment of drainage. B. CORRECT: Application of a sterile dressing will contain the drainage and allow continuous assessment of color and quantity. C. Application of direct pressure can cause discomfort and potential harm to the client. d. Place the client with the head of the bed elevated to promote lung expansion.

3. A patient receives atropine, an anticholinergic drug, in preparation for surgery. The nurse expects this drug to affect the gastrointestinal (GI) tract by which action? a. Increasing gastric emptying b. Relaxing pyloric and ileocecal sphincters c. Decreasing secretions and peristaltic action d. Stimulating the nervous system of the GI tract

3. c. The parasympathetic nervous system stimulates activity of the gastrointestinal (GI) tract, increasing motility and secretions and relaxing sphincters to promote movement of contents. A drug that blocks this activity decreases secretions and peristalsis, slows gastric emptying, and contracts sphincters. The enteric nervous system of the GI tract is modulated by sympathetic and parasympathetic influence.

4 A patient who has duodenal ulcers is receiving long-term therapy with ranitidine (Zantac). The nurse includes in the care plan that the patient should be monitored for which adverse effects? A. Photophobia and skin irritations B. Neutropenia and thrombocytopenia C. Dyspnea and productive coughing D. Urinary hesitation and fluid retention

4 Answer: 2 Rationale: Blood dyscrasias have been reported, especially neutropenia and thrombocytopenia, with long-term use. Periodic blood counts should be performed. Options 1, 3, and 4 are incorrect. Ranitidine does not cause photophobia and skin irritations. Dyspnea and productive cough are not expected adverse effects for this medication. Ranitidine is not known to cause these symptoms. Cognitive Level: Applying; Client Need: Physiological Integrity; Nursing Process: Planning

5 A health care provider orders magnesium hydroxide (Milk of Magnesia) for a patient with constipation, secondary to postoperative opioid use. Before administering the drug, the nurse would assess: A. Blood pressure. B. Dosage of the opioid drug prescribed. C. The patient's ability to ambulate to the bathroom. D. Bowel Sounds

5 Answer: 4 Rationale: Because magnesium hydroxide will stimulate peristalsis, it is important that the nurse assess for bowel sounds before giving the drug. If blockage or an ileus is suspected, the drug should be held and the provider notified. Options 1, 2, and 3 are incorrect. Blood pressure is an important vital sign to monitor postoperatively, but the magnesium hydroxide should not have direct effects. The dosage of the opioid drug and the patient's ability to ambulate to the bathroom will not impact the drug's use or action. Cognitive Level: Applying; Client Need: Physiological Integrity; Nursing Process: Assessment

5 The prescriber orders sucralfate (Carafate) for a patient with peptic ulcer disease. The nurse should question the order if the patient is concurrently taking: A. Proton pump inhibitors. B. Calcium salt antacids. C. H2-receptor antagonists. D. Aluminum salt antacids.

5 Answer: 4 Rationale: Concurrent use of aluminum salts and sucralfate poses a risk for aluminum toxicity. Options 1, 2, and 3 are incorrect. There is no risk of aluminum toxicity with PPIs, other antacids, or H2-receptor antagonists. Cognitive Level: Applying; Client Need: Physiological Integrity; Nursing Process: Implementation

6 A patient asks the nurse about giving an over-the-counter drug, bismuth subsalicylate (Pepto-Bismol), to treat a daughter's diarrhea. On which of the following will the nurse base the recommendation? Select all that apply. A. Cause of diarrhea B. Normal activity level C. Age D. Weight E. School schedule

6 Answer: 1, 3 Rationale: The nurse should explore possible causes for the diarrhea with the mother before making a recommendation because if diarrhea is caused by infections, slowing motility may allow the infection to increase. Salicylates, including bismuth subsalicylate, are contraindicated in children under the age of 19 because of an increased risk for Reye's syndrome. Options 2, 4, and 5 are incorrect. Activity level and weight are important growth and development parameters to assess but are unrelated to the drug's use. The school schedule would not have a direct impact on which drug is recommended. Cognitive Level: Applying; Client Need:

6 The nurse who is caring for a patient with gastroesophageal reflux disease should question the order for which drug? A. H2-receptor antagonists B. Proton pump inhibitors C. Antibiotics D. Antacids

6 Answer: 3 Rationale: Antibiotics have no role in the treatment of GERD although certain antibiotics are used in treating PUD to eradicate the H. pylori organism. Options 1, 2, and 4 are incorrect. H2- receptor antagonists and PPIs are used routinely to relieve symptoms of GERD. OTC antacids provide intermittent relief for mild cases. Cognitive Level: Applying; Client Need: Physiological Integrity; Nursing Process: Implementation

4. A nurse is completing an assessment of a client who has a gastric ulcer. Which of the following findings should the nurse expect? (select all that apply.) A. Client reports pain relieved by eating. B. Client states that pain often occurs at night. C. Client reports a sensation of bloating. d. Client states that pain occurs 30 min to 1 hr after a meal. e. Client experiences pain upon palpation of the epigastric region.

A. A client who has a duodenal ulcer will report that pain is relieved by eating. B. Pain that rarely occurs at night is an expected finding. C. CORRECT: A client report of a bloating sensation is an expected finding.d. CORRECT: A client who has a gastric ulcer will often report pain 30 to 60 min after a meal. e. CORRECT: Pain in the epigastric region upon palpation is an expected finding.

4. A nurse is completing discharge teaching with a client who has Crohn's disease. Which of the following instructions should the nurse include in the teaching? A. decrease intake of calorie‐dense foods. B. drink canned protein supplements. C. increase intake of high fiber foods. d. eat high‐residue foods.

A. A high‐protein diet is recommended for the client who has Crohn's disease. B. CORRECT: A high‐protein diet is recommended for the client who has Crohn's disease. Canned protein supplements are encouraged. C. A low‐fiber diet is recommended for the client who has Crohn's disease to reduce inflammation. d. instruct the client to eat low‐residue foods to reduce inflammation.

2. A nurse is providing discharge teaching to a client who has a new prescription for aluminum hydroxide. Which of the following information should the nurse include in the teaching? A. Take the medication with food. B. Monitor for diarrhea. C. Wait 1 hr before taking other oral medications. d. Maintain a low‐fiber diet.

A. Advise the client to take aluminum hydroxide on an empty stomach. B. include in the teaching that aluminum hydroxide can cause constipation. C. CORRECT: Advise the client not to take oral medications within 1 hr of an antacid. d. include in the teaching for the client to increase dietary fiber due to the constipating effect of the medication.

5. A nurse in a clinic is teaching a client who has ulcerative colitis. Which of the following statements by the client indicates understanding of the teaching? A. "i will plan to limit fiber in my diet." B. "i will restrict fluid intake during meals." C. "i will switch to black tea instead of drinking coffee." d. "i will try to eat cold foods rather than warm when my stomach feels upset.

A. CORRECT: A low‐fiber diet is recommended for the client who has ulcerative colitis to reduce inflammation. B. A client who has dumping syndrome should avoid fluids with meals. C. Caffeine can increase diarrhea and cramping. The client should avoid caffeinated beverages, such as black tea. d. The client should avoid cold foods because these can increase intestinal motility and cause exacerbation of manifestations.

1. A nurse is caring for a client who has a new diagnosis of gastroesophageal reflux disease (Gerd). The nurse should expect prescriptions for which of the following medications? (select all that apply.) A. Antacids B. Histamine2 receptor antagonists C. Opioid analgesics d. Fiber laxatives e. Protonpumpinhibitors

A. CORRECT: Antacids neutralize gastric acid which irritates the esophagus during reflux. B. CORRECT: Histamine2 receptor antagonists decrease acid secretion, which contributes to reflux. C. Opioid analgesics are not effective in treating Gerd. d. Fiber laxatives are not effective in treating Gerd.e. CORRECT: Proton pump inhibitors decrease gastric acid production, which contributes to reflux.

3. A nurse is caring for a client who has a small bowel obstruction from adhesions. Which of the following findings are consistent with this diagnosis? (select all that apply.) A. emesis greater than 500 mL with a fecal odor B. report of spasmodic abdominal pain C. High‐pitched bowel sounds d. Abdomen flat with rebound tenderness to palpation e. Laboratoryfindingsindicatingmetabolicacidosis

A. CORRECT: Large emesis with a fecal odor is a finding in a client who has a small bowel obstruction. B. CORRECT: report of abdominal pain is a finding in a client who has a small bowel obstruction. C. CORRECT: High‐pitched bowel sounds are a manifestation of a small‐ or large‐bowel obstruction. d. Abdominal distention is a finding in a client who has a small bowel obstruction. e. Metabolic alkalosis due to the loss of gastric acid is a finding in a client who has a small bowel obstruction.

5. A nurse is teaching a client who has a duodenal ulcer and a new prescription for esomeprazole. Which of the following information should the nurse include in the teaching? (select all that apply.) A. Take the medication 1 hr before a meal. B. Limit NsAids when taking this medication. C. expect skin flushing when taking this medication. d. increase fiber intake when taking this medication. e. Chew the medication thoroughly before swallowing.

A. CORRECT: Take the medication 1 hr before meals.B. CORRECT: Limit taking NsAids when on this medication. C. skin flushing is not an adverse effect of this medication. d. Fiber intake does not need to be increased when taking this medication.e. swallow the capsule whole. it should not be crushed or chewed.

5. A nurse is completing discharge teaching with a client who has irritable bowel syndrome (iBs). Which of the following instructions should the nurse include? A. Keep a food diary to identify triggers to exacerbation. B. Consume 15 to 20 g of fiber daily. C. Plan three moderate to large meals per day. d. Limit fluid intake to 1 L each day.

A. CORRECT: The client should keep a food diary to identify foods that trigger exacerbation of manifestations. B. The client should increase daily fiber intake to 30 to 40 g. C. The client should eat small frequent meals.d. The client should drink 2 to 3 L fluids per day to promote a consistent bowel pattern.

5. A nurse is caring for a client who is receiving TPN solution. The current bag of solution was hung 24 hr ago, and 400 mL remains to infuse. Which of the following is the appropriate action for the nurse to take? A. remove the current bag and hang a new bag. B. infuse the remaining solution at the current rate and then hang a new bag. C. increase the infusion rate so the remaining solution is administered within the hour and hang a new bag. d. remove the current bag and hang a bag of lactated ringer's.

A. CORRECT: The current bag of TPN should not hang more than 24 hr due to the risk of infection. B. A bag of TPN should not infuse for more than 24 hr due to the risk of infection. C. The rate of TPN infusion should never be increased abruptly due to the risk of hyperglycemia. d. Administration of TPN should never be discontinued abruptly. if the solution needs replacing and another bag is not available, use dextrose 10% in water to maintain blood glucose levels.

2. A nurse is planning care for a client who has a small bowel obstruction and a nasogastric (NG) tube in place. Which of the following interventions should the nurse include? (select all that apply.) A. document the NG drainage with the client's output. B. irrigate the NG tube every 8 hr. C. Assess bowel sounds. d. Provide oral hygiene every 2 hr. e. MonitorNGtubeforplacement.

A. CORRECT: document the NG drainage as output. This helps determine the amount of fluid replacement needed. B. The NG tube is irrigated every 4 hr to maintain patency. C. CORRECT: Bowel sounds should be assessed to evaluate treatment and resolution of the obstruction.d. CORRECT: An NG tube promotes mouth breathing. Provide frequent oral hygiene to provide comfort.e. CORRECT: Check the placement of the NG tube prior to irrigation to prevent aspiration and periodically to prevent an increase in abdominal distention.

4. A nurse is teaching a client who has a hiatal hernia. Which of the following client statements indicates an understanding of the teaching? A. "i can take my medications with soda." B. "Peppermint tea will increase my indigestion." C. "Wearing an abdominal binder will limit my manifestations." d. "i will drink hot chocolate at bedtime to help me sleep." e. "i can lift weights as a way to exercise."

A. Carbonated beverages decrease Les pressure and should be avoided by the client who has a hiatal hernia. B. CORRECT: Peppermint decreases Les pressure and should be avoided by the client who has a hiatal hernia. C. Tight restrictive clothing or abdominal binders should be avoided by the client who has a hiatal hernia, as this increases intra‐abdominal pressure and causes the protrusion of the stomach into the thoracic cavity. d. The client should avoid consuming anything immediately prior to bedtime. Additionally, chocolate relaxes the lower esophageal sphincter and should be avoided by a client who has a hiatal hernia. e. Heavy lifting and vigorous activities are to be avoided in the client who has a hiatal hernia.

2. A nurse is teaching a client who has a new diagnosis of dumping syndrome following gastric surgery. Which of the following information should the nurse include in the teaching? A. eat three moderate‐sized meals a day. B. drink at least one glass of water with each meal. C. eat a bedtime snack that contains a milk product. d. increase protein in the diet.

A. Consume small, frequent meals rather than moderate‐sized meals. B. eliminate liquids with meals and for 1 hr prior to and following meals. C. Avoid milk products.d. CORRECT: eat a high‐protein, high‐fat, low‐fiber, and moderate‐ to low‐carbohydrate diet.

3. A nurse is completing discharge teaching for a client who has an infection due to Helicobacter pylori (H. pylori). Which of the following statements by the client indicates understanding of the teaching? A. "i will continue my prescription for corticosteroids." B. "i will schedule a CT scan to monitor improvement." C. "i will take a combination of medications for treatment." d. "i will have my throat swabbed to recheck for this bacteria."

A. Corticosteroid use is a contributing factor to an infection caused by H. pylori. B. An esophagogastroduodenoscopy is done to evaluate for the presence of H. pylori and to evaluate effectiveness of treatment. C. CORRECT: A combination of antibiotics and a histamine2 receptor antagonist is used to treat an infection caused by H. pylori. d. H. pylori is evaluated by obtaining gastric samples, not a throat swab.

1. A nurse is reviewing the laboratory data of a client who has an acute exacerbation of Crohn's disease. Which of the following blood laboratory results should the nurse expect to be elevated? (select all that apply.) A. Hematocrit B. erythrocyte sedimentation rate C. WBC d. Folic acid e. Albumin

A. Hematocrit is decreased as a result of chronic blood loss. B. CORRECT: increased erythrocyte sedimentation rate is a finding in a client who has Crohn's disease as a result of inflammation.C. CORRECT: increased WBC is a finding in a client who has Crohn's disease.d. A decrease in folic acid level is indicative of malabsorption due to Crohn's disease. e. A decrease in albumin is indicative of malabsorption due to Crohn's disease.

3. A nurse is completing an assessment of a client who has Gerd. Which of the following is an expected finding? A. Absence of saliva B. Painful swallowing C. sweet taste in mouth d. Absence of eructation

A. Hypersalivation is an expected finding in a client who has Gerd. B. CORRECT: Painful swallowing is a manifestation of Gerd due to esophageal stricture or inflammation. C. A client who has Gerd would report a bitter taste in the mouth. d. increased burping is an expected finding in a client who has Gerd.

2. A nurse is admitting a client who has bleeding esophageal varices. The nurse should expect a prescription for which of the following medications? A. Propranolol B. Metoclopramide C. Famotidine d. Vasopressin

A. Propranolol is not used for clients who are actively bleeding. it can be given prophylactically to decrease portal hypertension. B. Metoclopramide decreases motility of the esophagus and stomach. C. Histamine2‐receptor antagonists are administered following surgical procedures for bleeding esophageal varices. d. CORRECT: Vasopressin constricts blood vessels and is used to treat bleeding esophageal varices.

2. A nurse is assessing a client who has been taking prednisone following an exacerbation of inflammatory bowel disease. The nurse should recognize which of the following findings as the priority? A. Client reports difficulty sleeping. B. The client's urine is positive for glucose. C. Client reports having an elevated body temperature. d. Client reports gaining 4 lb in the last 6 months.

A. The client is at risk for sleep deprivation because prednisone can cause anxiety and insomnia. However, another finding is the priority. B. The client is at risk for hyperglycemia because prednisone can cause glucose intolerance. However, another finding is the priority. C. CORRECT: The greatest risk to the client is infection because prednisone can cause immunosuppression. Therefore, identify manifestations of an infection, such as an elevated body temperature, as the priority finding. d. The client is at risk for weight gain because prednisone can cause fluid retention. However, another finding is the priority.

5. A nurse is completing discharge teaching with a client who is postoperative following fundoplication. Which of the following statements by the client indicates understanding of the teaching? A. "When sitting in my lounge chair after a meal, i will lower the back of it." B. "i will try to eat three large meals a day." C. "i will elevate the head of my bed on blocks." d. "i will avoid eating within 1 hour before bedtime."

A. The client is instructed to remain upright after eating following a fundoplication. B. The client is instructed to avoid large meals after a fundoplication. C. CORRECT: After a fundoplication, the client is instructed to elevate the head of the bed to limit reflux. d The client should avoid eating 2 hr before bedtime to reduce the risk for reflux.

3. A nurse is teaching a client who has a new prescription for sulfasalazine. Which of the following instructions should the nurse include in the teaching? A. "Take the medication 2 hours after eating." B. "discontinue this medication if your skin turns yellow‐orange." C. "Notify the provider if you experience a sore throat." d. "expect your stools to turn black."

A. sulfasalazine should be taken right after meals and with a full glass of water to reduce gastric upset and prevent crystalluria. B. yellow‐orange coloring of the skin and urine is a harmless effect of sulfasalazine. C. CORRECT: sulfasalazine can cause blood dyscrasias. The client should monitor and report any manifestations of infection, such as a sore throat. d. sulfasalazine can cause thrombocytopenia and bleeding. Black stools are a manifestation of gastrointestinal bleeding, and the client should report this to the provider.

50. Which menu choice by the patient with diverticulosis is best for preventing diverticulitis? a. Navy bean soup and vegetable salad b. Whole grain pasta with tomato sauce c. Baked potato with low-fat sour cream d. Roast beef sandwich on whole wheat bread

ANS: A A diet high in fiber and low in fats and red meat is recommended to prevent diverticulitis. Although all of the choices have some fiber, the bean soup and salad will be the highest in fiber and the lowest in fat.

25. The nurse admitting a patient with acute diverticulitis explains that the initial plan of care is to a. administer IV fluids. b. prepare for colonoscopy. c. give stool softeners and enemas. d. order a diet high in fiber and fluids.

ANS: A A patient with acute diverticulitis will be NPO and given parenteral fluids. A diet high in fiber and fluids will be implemented before discharge. Bulk-forming laxatives, rather than stool softeners, are usually given, and these will be implemented later in the hospitalization. The patient with acute diverticulitis will not have enemas or a colonoscopy because of the risk for perforation and peritonitis.

36. Which patient should the nurse assess first after receiving change-of-shift report? a. A patient with esophageal varices who has a rapid heart rate b. A patient with a history of gastrointestinal bleeding who has melena c. A patient with nausea who has a dose of metoclopramide (Reglan) due d. A patient who is crying after receiving a diagnosis of esophageal cancer

ANS: A A patient with esophageal varices and a rapid heart rate indicate possible hemodynamic instability caused by GI bleeding. The other patients do not indicate acutely life-threatening complications.

5. A patient complains of gas pains and abdominal distention 2 days after a small bowel resection. Which nursing action should the nurse take? a. Encourage the patient to ambulate. b. Instill a mineral oil retention enema. c. Administer the prescribed IV morphine sulfate. d. Offer the prescribed promethazine (Phenergan).

ANS: A Ambulation will improve peristalsis and help the patient eliminate flatus and reduce gas pain. A mineral oil retention enema is helpful for constipation with hard stool. A return-flow enema might be used to relieve persistent gas pains. Morphine will further reduce peristalsis. Promethazine is used as an antiemetic rather than to decrease gas pains or distention.

8. Which nursing action will be included in the plan of care for a 25-yr-old male patient with a new diagnosis of irritable bowel syndrome (IBS)? a. Encourage the patient to express concerns and ask questions about IBS. b. Suggest that the patient increase the intake of milk and other dairy products. c. Teach the patient to avoid using nonsteroidal antiinflammatory drugs (NSAIDs). d. Teach the patient about the use of alosetron (Lotronex) to reduce IBS symptoms.

ANS: A Because psychologic and emotional factors can affect the symptoms for IBS, encouraging the patient to discuss emotions and ask questions is an important intervention. Alosetron has serious side effects and is used only for female patients who have not responded to other therapies. Although yogurt may be beneficial, milk is avoided because lactose intolerance can contribute to symptoms in some patients. NSAIDs can be used by patients with IBS.

1. Which finding for a young adult who follows a vegan diet may indicate the need for cobalamin supplementation? a. Paresthesias b. Ecchymoses c. Dry, scaly skin d. Gingival swelling

ANS: A Cobalamin (vitamin B12) cannot be obtained from foods of plant origin, so the patient will be most at risk for signs of cobalamin deficiency, such as paresthesias, peripheral neuropathy, and anemia. The other symptoms listed are associated with other nutritional deficiencies but would not be associated with a vegan diet.

19. What diagnostic test should the nurse anticipate for an older patient who is vomiting "coffee-ground" emesis? a. Endoscopy b. Angiography c. Barium studies d. Gastric analysis

ANS: A Endoscopy is the primary tool for visualization and diagnosis of upper gastrointestinal (GI) bleeding. Angiography is used only when endoscopy cannot be done because it is more invasive and has more possible complications. Barium studies are helpful in determining the presence of gastric lesions, but not whether the lesions are actively bleeding. Gastric analysis testing may help with determining the cause of gastric irritation, but it is not used for acute GI bleeding.

5. The nurse receives the following information about a patient who is scheduled for a colonoscopy. Which information should be communicated to the health care provider before sending the patient for the procedure? a. The patient declined to drink the prescribed laxative solution. b. The patient has had an allergic reaction to shellfish and iodine. c. The patient has a permanent pacemaker to prevent bradycardia. d. The patient is worried about discomfort during the examination.

ANS: A If the patient has had inadequate bowel preparation, the colon cannot be visualized and the procedure would be rescheduled. Because contrast solution is not used during colonoscopy, the iodine allergy is not pertinent. A pacemaker is a contraindication to magnetic resonance imaging but not to colonoscopy. The nurse should instruct the patient about the sedation used during the examination to decrease the patient's anxiety about discomfort.

5. A patient who has just been started on enteral nutrition of full-strength formula at 100 mL/hr has 6 liquid stools the first day. Which action should the nurse plan to take? a. Slow the infusion rate of the feeding. b. Check gastric residual volumes more often. c. Change the enteral feeding system and formula every 8 hours. d. Discontinue administration of water through the feeding tube.

ANS: A Loose stools indicate poor absorption of nutrients and indicate a need to slow the feeding rate or decrease the concentration of the feeding. Water should be given when patients receive enteral feedings to prevent dehydration. When a closed enteral feeding system is used, the tubing and formula are changed every 24 hours. High residual volumes do not contribute to diarrhea.

15. A patient has a new diagnosis of Crohn's disease after having frequent diarrhea and a weight loss of 10 lb (4.5 kg) over 2 months. The nurse will plan to teach about a. medication use. b. fluid restriction. c. enteral nutrition. d. activity restrictions.

ANS: A Medications are used to induce and maintain remission in patients with inflammatory bowel disease (IBD). Decreased activity level is indicated only if the patient has severe fatigue and weakness. Fluids are needed to prevent dehydration. There is no advantage to enteral feedings.

54. Which prescribed intervention for a patient with chronic short bowel syndrome will the nurse question? a. Senna 1 tablet every day b. Ferrous sulfate 325 mg daily c. Psyllium (Metamucil) 3 times daily d. Diphenoxylate with atropine (Lomotil) prn loose stools

ANS: A Patients with short bowel syndrome have diarrhea because of decreased nutrient and fluid absorption and would not need stimulant laxatives. Iron supplements are used to prevent iron-deficiency anemia, bulk-forming laxatives help make stools less watery, and opioid antidiarrheal drugs are helpful in slowing intestinal transit time.

44. A patient with Crohn's disease who is taking infliximab (Remicade) calls the nurse in the outpatient clinic about new symptoms. Which symptom is most important to communicate to the health care provider? a. Fever b. Nausea c. Joint pain d. Headache

ANS: A Since infliximab suppresses the immune response, rapid treatment of infection is essential. The other patient complaints are common side effects of the medication, but they do not indicate any potentially life-threatening complications.

47. A patient is awaiting surgery for acute peritonitis. Which action will the nurse include in the plan of care? a. Position patient with the knees flexed. b. Avoid use of opioids or sedative drugs. c. Offer frequent small sips of clear liquids. d. Assist patient to breathe deeply and cough.

ANS: A There is less peritoneal irritation with the knees flexed, which will help decrease pain. Opioids and sedatives are typically given to control pain and anxiety. Preoperative patients with peritonitis are given IV fluids for hydration. Deep breathing and coughing will increase the patient's discomfort.

7. A malnourished patient is receiving a parenteral nutrition (PN) infusion containing amino acids and dextrose from a bag that was hung with a new tubing and filter 24 hours ago. The nurse observes that about 50 mL remain in the PN container. Which action should the nurse take? a. Add a new container of PN using the current tubing and filter. b. Hang a new container of PN and change the IV tubing and filter. c. Infuse the remaining 50 mL and then hang a new container of PN. d. Ask the health care provider to clarify the written PN prescription.

ANS: B All PN solutions and tubings are changed at 24 hours. Infusion of the additional 50 mL will increase patient risk for infection. The nurse (not the health care provider) is responsible for knowing the indicated times for tubing and filter changes.

52. The nurse is admitting a 67-yr-old patient with new-onset steatorrhea. Which question is most important for the nurse to ask? a. "How much milk do you usually drink?" b. "Have you noticed a recent weight loss?" c. "What time of day do your bowels move?" d. "Do you eat meat or other animal products?"

ANS: B Although all of the questions provide useful information, it is most important to determine if the patient has an imbalance in nutrition because of the steatorrhea.

32. The nurse is assessing a patient who had a total gastrectomy 8 hours ago. What information is most important to report to the health care provider? a. Hemoglobin (Hgb) 10.8 g/dL b. Temperature 102.1° F (38.9° C) c. Absent bowel sounds in all quadrants d. Scant nasogastric (NG) tube drainage

ANS: B An elevation in temperature may indicate leakage at the anastomosis, which may require return to surgery or keeping the patient NPO. The other findings are expected in the immediate postoperative period for patients who have this surgery and do not require any urgent action.

9. A patient being admitted with an acute exacerbation of ulcerative colitis reports crampy abdominal pain and passing 15 or more bloody stools a day. The nurse will plan to a. administer IV metoclopramide (Reglan). b. discontinue the patient's oral food intake. c. administer cobalamin (vitamin B12) injections. d. teach the patient about total colectomy surgery.

ANS: B An initial therapy for an acute exacerbation of inflammatory bowel disease (IBD) is to rest the bowel by making the patient NPO. Metoclopramide increases peristalsis and will worsen symptoms. Cobalamin (vitamin B12) is absorbed in the ileum, which is not affected by ulcerative colitis. Although total colectomy is needed for some patients, there is no indication that this patient is a candidate.

10. Which nursing action will the nurse include in the plan of care for a 35-yr-old male patient admitted with an exacerbation of inflammatory bowel disease (IBD)? a. Restrict oral fluid intake. b. Monitor stools for blood. c. Ambulate six times daily. d. Increase dietary fiber intake.

ANS: B Because anemia or hemorrhage may occur with IBD, stools should be assessed for the presence of blood. The other actions would not be appropriate for the patient with IBD. Dietary fiber may increase gastrointestinal motility and exacerbate the diarrhea, severe fatigue is common with IBD exacerbations, and dehydration may occur.

34. A patient in the emergency department has just been diagnosed with peritonitis caused by a ruptured diverticulum. Which prescribed intervention will the nurse implement first? a. Insert a urinary catheter to drainage. b. Infuse metronidazole (Flagyl) 500 mg IV. c. Send the patient for a computerized tomography scan. d. Place a nasogastric (NG) tube to intermittent low suction.

ANS: B Because peritonitis can be fatal if treatment is delayed, the initial action should be to start antibiotic therapy (after any ordered cultures are obtained). The other actions can be done after antibiotic therapy is initiated.

40. Which information obtained by the nurse interviewing a 30-yr-old male patient is most important to communicate to the health care provider? a. The patient has a history of constipation. b. The patient has noticed blood in the stools. c. The patient had an appendectomy at age 27. d. The patient smokes a pack/day of cigarettes.

ANS: B Blood in the stools is a possible clinical manifestation of colorectal cancer and requires further assessment by the health care provider. The other patient information will also be communicated to the health care provider, but does not indicate an urgent need for further testing or intervention.

18. A patient hospitalized with chronic heart failure eats only about 50% of each meal and reports "feeling too tired to eat." Which action should the nurse take first? a. Teach the patient about the importance of good nutrition. b. Serve multiple small feedings of high-calorie, high-protein foods. c. Consult with the health care provider about parenteral nutrition (PN). d. Obtain an order for enteral feedings of liquid nutritional supplements.

ANS: B Eating small amounts of food frequently throughout the day is less fatiguing and will improve the patient's ability to take in more nutrients. Teaching the patient may be appropriate but will not address the patient's inability to eat more because of fatigue. Enteral nutrition or PN may be needed if the patient is unable to take in enough nutrients orally but increasing the oral intake should be attempted first.

11. What action should the nurse take when caring for a patient with a soft, silicone nasogastric tube in place for enteral nutrition? a. Avoid giving medications through the feeding tube. b. Keep head of bed elevated to 30- to 45-degree angle. c. Replace the tube every 3 days to avoid mucosal damage. d. Administer medications mixed with enteral feeding formula.

ANS: B Elevate the head of the bed to decrease the risk of aspiration. The tubes are less likely to cause mucosal damage than the stiffer polyvinyl chloride tubes used for nasogastric suction and do not need to be replaced at certain intervals. Medications can be given through these tubes but flushing after medication administration is important to avoid clogging. Do not mix medications with formula, as the combination can clog the tube.

12. Which information will the nurse provide for a patient with newly diagnosed gastroesophageal reflux disease (GERD)? a. "Peppermint tea may reduce your symptoms." b. "Keep the head of your bed elevated on blocks." c. "You should avoid eating between meals to reduce acid secretion." d. "Vigorous physical activities may increase the incidence of reflux."

ANS: B Elevating the head of the bed will reduce the incidence of reflux while the patient is sleeping. Peppermint will decrease lower esophageal sphincter (LES) pressure and increase the chance for reflux. Small, frequent meals are recommended to avoid abdominal distention. There is no need to make changes in physical activities because of GERDD

14. After a total proctocolectomy and permanent ileostomy, the patient tells the nurse, "I cannot manage all these changes. I don't want to look at the stoma." What is the best action by the nurse? a. Reassure the patient that ileostomy care will become easier. b. Ask the patient about the concerns with stoma management. c. Postpone any teaching until the patient adjusts to the ileostomy. d. Develop a detailed written list of ostomy care tasks for the patient.

ANS: B Encouraging the patient to share concerns assists in helping the patient adjust to the body changes. Acknowledgment of the patient's feelings and concerns is important rather than offering false reassurance. Because the patient indicates that the feelings about the ostomy are the reason for the difficulty with the many changes, development of a detailed ostomy care plan will not improve the patient's ability to manage the ostomy. Although detailed ostomy teaching may be postponed, the nurse should offer teaching about some aspects of living with an ostomy.

22. A young adult patient is hospitalized with massive abdominal trauma from a motor vehicle crash. The patient asks the nurse about the purpose of receiving famotidine (Pepcid). What should the nurse explain about the action of the medication? a. "It decreases nausea and vomiting." b. "It inhibits development of stress ulcers." c. "It lowers the risk for H. pylori infection." d. "It prevents aspiration of gastric contents."

ANS: B Famotidine is administered to prevent the development of physiologic stress ulcers, which are associated with a major physiologic insult such as massive trauma. Famotidine does not decrease nausea or vomiting, prevent aspiration, or prevent H. pylori infection.

32. A critically ill patient with sepsis is frequently incontinent of watery stools. What action by the nurse will prevent complications associated with ongoing incontinence? a. Apply incontinence briefs. b. Use a fecal management system c. Insert a rectal tube with a drainage bag. d. Assist the patient to a commode frequently.

ANS: B Fecal management systems are designed to contain loose stools and can be in place for as long as 4 weeks without causing damage to the rectum or anal sphincters. Although incontinence briefs may be helpful, unless they are changed frequently, they are likely to increase the risk for skin breakdown. Rectal tubes are avoided because of possible damage to the anal sphincter and ulceration of the rectal mucosa. A critically ill patient will not be able to tolerate getting up frequently to use the commode or bathroom.

16. A young woman who has Crohn's disease develops a fever and symptoms of a urinary tract infection (UTI) with tan, fecal-smelling urine. What information will the nurse add to a general teaching plan about UTIs in order to individualize the teaching for this patient? a. Bacteria in the perianal area can enter the urethra. b. Fistulas can form between the bowel and bladder. c. Drink adequate fluids to maintain normal hydration. d. Empty the bladder before and after sexual intercourse.

ANS: B Fistulas between the bowel and bladder occur in Crohn's disease and can lead to UTI. Teaching for UTI prevention in general includes good hygiene, adequate fluid intake, and voiding before and after intercourse.

6. A patient who has gastroesophageal reflux disease (GERD) is experiencing increasing discomfort. Which patient statement to the nurse indicates that additional teaching about GERD is needed? a. "I quit smoking years ago, but I chew gum." b. "I eat small meals and have a bedtime snack." c. "I take antacids between meals and at bedtime each night." d. "I sleep with the head of the bed elevated on 4-inch blocks."

ANS: B GERD is exacerbated by eating late at night, and the nurse should plan to teach the patient to avoid eating at bedtime. The other patient actions are appropriate to control symptoms of GERD.

23. Which information will the nurse include in teaching a patient who had a proctocolectomy and ileostomy for ulcerative colitis? a. Restrict fluid intake to prevent constant liquid drainage from the stoma. b. Use care when eating high-fiber foods to avoid obstruction of the ileum. c. Irrigate the ileostomy daily to avoid having to wear a drainage appliance. d. Change the pouch every day to prevent leakage of contents onto the skin.

ANS: B High-fiber foods are introduced gradually and should be well chewed to avoid obstruction of the ileostomy. Patients with ileostomies lose the absorption of water in the colon and need to take in increased amounts of fluid. The pouch should be drained frequently but is changed every 5 to 7 days. The drainage from an ileostomy is liquid and continuous, so control by irrigation is not possible.

15. Which action for a patient receiving enteral nutrition through a percutaneous endoscopic gastrostomy (PEG) may be delegated to a licensed practical/vocational nurse (LPN/VN)? a. Assessing the patient's nutritional status weekly b. Providing skin care to the area around the tube site c. Teaching the patient how to administer the feedings d. Determining the need for adding water to the feedings

ANS: B LPN/VN education and scope of practice include actions such as dressing changes and wound care. Patient teaching and complex assessments (such as patient nutrition and hydration status) require registered nurse (RN)-level education and scope of practice.

51. After change-of-shift report, which patient should the nurse assess first? a. A 40-yr-old male patient with celiac disease who has frequent frothy diarrhea b. A 30-yr-old female patient with a femoral hernia who has abdominal pain and vomiting c. A 30-yr-old male patient with ulcerative colitis who has severe perianal skin breakdown d. A 40-yr-old female patient with a colostomy bag that is pulling away from the adhesive wafer

ANS: B Pain and vomiting with a femoral hernia suggest possible strangulation, which will necessitate emergency surgery. The other patients have less urgent problems.

6. A 58-yr-old patient with blunt abdominal trauma from a motor vehicle crash undergoes peritoneal lavage. If the lavage returns brown fecal drainage, which action will the nurse plan to take next? a. Auscultate the bowel sounds. b. Prepare the patient for surgery. c. Check the patient's oral temperature. d. Obtain information about the accident.

ANS: B Return of brown drainage and fecal material suggests perforation of the bowel and the need for immediate surgery. Auscultation of bowel sounds, checking the temperature, and obtaining information about the accident are appropriate actions, but the priority is to prepare to send the patient for emergency surgery.

11. Which patient statement indicates that the nurse's teaching about sulfasalazine (Azulfidine) for ulcerative colitis has been effective? a. "The medication will be tapered if I need surgery." b. "I will need to use a sunscreen when I am outdoors." c. "I will need to avoid contact with people who are sick." d. "The medication prevents the infections that cause diarrhea."

ANS: B Sulfasalazine may cause photosensitivity in some patients. It is not used to treat infections. Sulfasalazine does not reduce immune function. Unlike corticosteroids, tapering of sulfasalazine is not needed.

29. Which information about dietary management should the nurse include when teaching a patient with peptic ulcer disease (PUD)? a. "You will need to remain on a bland diet." b. "Avoid foods that cause pain after you eat them." c. "High-protein foods are least likely to cause pain." d. "You should avoid eating any raw fruits and vegetables."

ANS: B The best information is that each person should choose foods that are not associated with postprandial discomfort. Raw fruits and vegetables may irritate the gastric mucosa but chewing well seems to decrease this problem and some patients may tolerate these foods well. High-protein foods help neutralize acid, but they also stimulate hydrochloric (HCl) acid secretion and may increase discomfort for some patients. Bland diets may be recommended during an acute exacerbation of PUD, but there is little evidence to support their use.

16. The nurse is preparing to teach a frail 79-yr-old Hispanic man who lives with an adult daughter about ways to improve nutrition. Which action should the nurse take first? a. Ask the daughter about the patient's food preferences. b. Determine who shops for groceries and prepares the meals. c. Question the patient about how many meals per day are eaten. d. Assure the patient that culturally preferred foods will be included.

ANS: B The family member who shops for groceries and cooks will be in control of the patient's diet, so the nurse will need to ensure that this family member is involved in any teaching or discussion about the patient's nutritional needs. The other information will also be assessed and used but will not be useful in meeting the patient's nutritional needs unless nutritionally appropriate foods are purchased and prepared.

12. A patient is receiving continuous enteral nutrition through a small-bore silicone feeding tube. What should the nurse plan for when this patient has a computed tomography (CT) scan ordered? a. Ask the health care provider to reschedule the scan. b. Shut the feeding off 30 to 60 minutes before the scan. c. Connect the feeding tube to continuous suction before and during the scan. d. Send a suction catheter with the patient in case of aspiration during the scan.

ANS: B The feeding should be shut off 30 to 60 minutes before any procedure requiring the patient to lie flat. Because the CT scan is ordered for diagnosis of patient problems, rescheduling is not usually an option. Prevention, rather than treatment, of aspiration is needed. Small-bore feeding tubes are soft and collapse easily with aspiration or suction, making nasogastric suction of gastric contents unreliable.

35. A 25-yr-old male patient calls the clinic complaining of diarrhea for 24 hours. Which action should the nurse take first? a. Inform the patient that laboratory testing of blood and stools will be necessary. b. Ask the patient to describe the character of the stools and any associated symptoms. c. Suggest that the patient drink clear liquid fluids with electrolytes, such as Gatorade or Pedialyte. d. Advise the patient to use over-the-counter loperamide (Imodium) to slow gastrointestinal (GI) motility.

ANS: B The initial response by the nurse should be further assessment of the patient. The other responses may be appropriate, depending on what is learned in the assessment.

17. After change-of-shift report, which patient will the nurse assess first? a. A 40-yr-old woman whose parenteral nutrition infusion bag has 30 minutes of solution left b. A 40-yr-old man with continuous enteral feedings who has developed pulmonary crackles c. A 30-yr-old man with 4+ generalized pitting edema and severe protein-calorie malnutrition d. A 30-yr-old woman whose gastrostomy tube is plugged after crushed medications were administered

ANS: B The patient data suggest aspiration may have occurred, and rapid assessment and intervention are needed. The other patients should also be assessed soon, but the data about them do not suggest any immediately life-threatening complications.

43. The nurse and a licensed practical/vocational nurse (LPN/VN) are working together to care for a patient who had an esophagectomy 2 days ago. Which action by the LPN/VN requires that the nurse intervene? a. The LPN/VN uses soft swabs to provide oral care. b. The LPN/VN positions the head of the bed in the flat position. c. The LPN/VN includes the enteral feeding volume when calculating intake. d. The LPN/VN encourages the patient to use pain medications before coughing.

ANS: B The patient's bed should be in Fowler's position to prevent reflux and aspiration of gastric contents. The other actions by the LPN/LVN are appropriate.

40. The nurse is administering IV fluid boluses and nasogastric irrigation to a patient with acute gastrointestinal (GI) bleeding. Which assessment finding is most important for the nurse to communicate to the health care provider? a. The bowel sounds are hyperactive in all four quadrants. b. The patient's lungs have crackles audible to the midchest. c. The nasogastric (NG) suction is returning coffee-ground material. d. The patient's blood pressure (BP) has increased to 142/84 mm Hg.

ANS: B The patient's lung sounds indicate that pulmonary edema may be developing because of the rapid infusion of IV fluid and that the fluid infusion rate should be slowed. The return of coffee-ground material in an NG tube is expected for a patient with upper GI bleeding. The BP is slightly elevated but would not be an indication to contact the health care provider immediately. Hyperactive bowel sounds are common when a patient has GI bleeding.

6. A young adult with extensive facial injuries from a motor vehicle crash is receiving continuous enteral nutrition through a percutaneous endoscopic gastrostomy (PEG). Which action will the nurse include in the plan of care? a. Keep the patient positioned lying on the left side. b. Flush the tube with 30 mL of water every 4 hours. c. Crush and mix medications in with the feeding formula. d. Obtain a daily abdominal radiograph to verify tube placement.

ANS: B The tube is flushed every 4 hours during continuous feedings to avoid tube obstruction. The patient should be positioned with the head of the bed elevated. Crushed medications mixed in with the formula are likely to clog the tube. An x-ray is obtained immediately after placement of the PEG tube to check position, but daily x-rays are not needed.

19. A patient's peripheral parenteral nutrition (PN) bag is nearly empty, and a new PN bag has not arrived yet from the pharmacy. Which intervention by the nurse is appropriate? a. Monitor the patient's capillary blood glucose every 6 hours. b. Infuse 5% dextrose in water until a new PN bag is delivered. c. Decrease the PN infusion rate to 10 mL/hr until a new bag arrives. d. Flush the peripheral line with saline until a new PN bag is available.

ANS: B To prevent hypoglycemia, the nurse should infuse a 5% dextrose solution until the next peripheral PN bag can be started. Decreasing the rate of the ordered PN infusion is beyond the nurse's scope of practice. Flushing the line and then waiting for the next bag may lead to hypoglycemia. Monitoring the capillary blood glucose every 6 hours would not identify hypoglycemia while awaiting the new PN bag.

12. A 22-yr-old female patient with an exacerbation of ulcerative colitis is having 15 to 20 stools daily and has excoriated perianal skin. Which patient behavior indicates that teaching regarding maintenance of skin integrity has been effective? a. The patient uses incontinence briefs to contain loose stools. b. The patient uses witch hazel compresses to soothe irritation. c. The patient asks for antidiarrheal medication after each stool. d. The patient cleans the perianal area with soap after each stool.

ANS: B Witch hazel compresses are suggested to reduce anal irritation and discomfort. Incontinence briefs may trap diarrhea and increase the incidence of skin breakdown. Antidiarrheal medications are not given 15 to 20 times a day. The perianal area should be washed with plain water or pH balanced cleanser after each stool.

4. The nurse is caring for a patient with an obstructed common bile duct. What condition should the nurse expect? a. Melena b. Steatorrhea c. Decreased serum cholesterol level d. Increased serum indirect bilirubin level

ANS: B A common bile duct obstruction will reduce the absorption of fat in the small intestine, leading to fatty stools. Gastrointestinal bleeding is not caused by common bile duct obstruction. Serum cholesterol levels are increased with biliary obstruction. Direct bilirubin level is increased with biliary obstruction.

12. A patient has just arrived in the recovery area after an upper endoscopy. Which information collected by the nurse is most important to communicate to the health care provider? a. The patient is very drowsy. b. The patient reports a sore throat. c. The oral temperature is 101.4° F. d. The apical pulse is 100 beats/min.

ANS: C A temperature elevation may indicate that an acute perforation has occurred. The other assessment data are normal immediately after the procedure.

8. Which finding by the nurse during abdominal auscultation indicates a need for a focused abdominal assessment? a. Loud gurgles b. High-pitched gurgles c. Absent bowel sounds d. Frequent clicking sounds

ANS: C Absent bowel sounds are abnormal and require further assessment by the nurse. The other sounds may be heard normally.

39. Which prescribed action will the nurse implement first for a patient who has vomited 1100 mL of blood? a. Give an IV H2 receptor antagonist. b. Draw blood for type and crossmatch. c. Administer 1 L of lactated Ringer's solution. d. Insert a nasogastric (NG) tube and connect to suction.

ANS: C Because the patient has vomited a large amount of blood, correction of hypovolemia and prevention of hypovolemic shock are the priorities. The other actions also are important to implement quickly but are not the highest priorities.

1. A 53-yr-old male patient with deep partial-thickness burns from a chemical spill in the workplace has severe pain followed by nausea during dressing changes. Which action will be most useful in decreasing the patient's nausea? a. Keep the patient NPO for 2 hours before dressing changes. b. Give the prescribed prochlorperazine before dressing changes. c. Administer prescribed morphine sulfate before dressing changes. d. Avoid performing dressing changes close to the patient's mealtimes.

ANS: C Because the patient's nausea is associated with severe pain, it is likely that it is precipitated by stress and pain. The best treatment will be to provide adequate pain medication before dressing changes. The nurse should avoid doing painful procedures close to mealtimes, but nausea or vomiting that occurs at other times also should be addressed. Keeping the patient NPO does not address the reason for the nausea and vomiting and will have an adverse effect on the patient's nutrition. Administration of antiemetics is not the best choice for a patient with nausea caused by pain. However, an antiemetic may be added later if the nausea persists despite pain management.

25. Which patient statement indicates that the nurse's postoperative teaching after a gastroduodenostomy has been effective? a. "I will drink more liquids with my meals." b. "I should choose high carbohydrate foods." c. "Vitamin supplements may prevent anemia." d. "Persistent heartburn is common after surgery."

ANS: C Cobalamin deficiency may occur after partial gastrectomy, and the patient may need toreceive cobalamin via injections or nasal spray. Although peptic ulcer disease may recur, persistent heartburn is not expected after surgery, and the patient should call the health care provider if this occurs. Ingestion of liquids with meals is avoided to prevent dumping syndrome. Foods that have moderate fat and low carbohydrate should be chosen to prevent dumping syndrome.

37. A patient returned from a laparoscopic Nissen fundoplication for hiatal hernia 4 hours ago. Which assessment finding is most important for the nurse to address immediately? a. The patient reports 7/10 (0 to 10 scale) abdominal pain. b. The patient is experiencing intermittent waves of nausea. c. The patient has no breath sounds in the left anterior chest. d. The patient has hypoactive bowel sounds in all four quadrants.

ANS: C Decreased breath sounds on one side may indicate a pneumothorax, which requires rapid diagnosis and treatment. The nausea and abdominal pain should be addressed, but they are not as high priority as the patient's respiratory status. Decreased bowel sounds are expected aftersurgery and require ongoing monitoring but no other action.

39. Which activity in the care of a patient with a new colostomy could the nurse delegate to unlicensed assistive personnel (UAP)? a. Document the appearance of the stoma. b. Place a pouching system over the ostomy. c. Drain and measure the output from the ostomy. d. Check the skin around the stoma for breakdown.

ANS: C Draining and measuring the output from the ostomy is included in UAP education and scope of practice. The other actions should be implemented by LPNs or RNs.

13. Which diet choice by the patient with an acute exacerbation of inflammatory bowel disease (IBD) indicates a need for more teaching? a. Scrambled eggs b. White toast and jam c. Oatmeal with cream d. Pancakes with syrup

ANS: C During acute exacerbations of IBD, the patient should avoid high-fiber foods such as whole grains. High-fat foods also may cause diarrhea in some patients. The other choices are low residue and would be appropriate for this patient.

2. A 76-yr-old woman with a body mass index (BMI) of 17 kg/m2 and a low serum albumin level is being admitted. Which assessment finding will the nurse expect? a. Restlessness b. Hypertension c. Pitting edema d. Food allergies

ANS: C Edema occurs when serum albumin levels and plasma oncotic pressure decrease. The blood pressure and level of consciousness are not directly affected by malnutrition. Food allergies are not an indicator of nutritional status.

9. Which patient choice for a snack 3 hours before bedtime indicates that the nurse's teaching about gastroesophageal reflux disease (GERD) has been effective? a. Chocolate pudding b. Glass of low-fat milk c. Cherry gelatin with fruit d. Peanut butter and jelly sandwich

ANS: C Gelatin and fruit are low fat and will not decrease lower esophageal sphincter (LES) pressure.Foods such as chocolate are avoided because they lower LES pressure. Milk products increase gastric acid secretion. High-fat foods such as peanut butter decrease both gastric emptying and LES pressure.

8. A patient's capillary blood glucose level is 120 mg/dL 6 hours after the nurse initiated a parenteral nutrition (PN) infusion. What is the appropriate action by the nurse? a. Obtain a venous blood glucose specimen. b. Slow the infusion rate of the PN infusion. c. Recheck the capillary blood glucose level in 4 to 6 hours. d. Contact the health care provider for infusion rate changes.

ANS: C Mild hyperglycemia is expected during the first few days after PN is started and requires ongoing monitoring. Because the glucose elevation is small and expected, infusion rate changes are not needed. There is no need to obtain a venous specimen for comparison. Slowing the rate of the infusion is beyond the nurse's scope of practice and will decrease the patient's nutritional intake.

38. A 19-yr-old woman is brought to the emergency department with a knife handle protruding from her abdomen. During the initial assessment of the patient, the nurse should a. remove the knife and assess the wound. b. determine the presence of Rovsing sign. c. check for circulation and tissue perfusion. d. insert a urinary catheter and assess for hematuria.

ANS: C The initial assessment is focused on determining whether the patient has hypovolemic shock. The knife should not be removed until the patient is in surgery, where bleeding can be controlled. Rovsing sign is assessed in the patient with suspected appendicitis. Assessment for bladder trauma is not part of the initial assessment.

14. A 20-yr-old woman is being admitted with electrolyte disorders of unknown etiology. Which assessment finding is most important to report to the health care provider? a. The patient uses laxatives daily. b. The patient's knuckles are macerated. c. The patient's serum potassium level is 2.9 mEq/L. d. The patient has a history of extreme weight fluctuations.

ANS: C The low serum potassium level may cause life-threatening cardiac dysrhythmias, and potassium supplementation is needed rapidly. The other information will also be reported because it suggests that bulimia may be the etiology of the patient's electrolyte disturbances, but it does not suggest imminent life-threatening complications.

26. At his first postoperative checkup appointment after a gastrojejunostomy (Billroth II), a patient reports that dizziness, weakness, and palpitations occur about 20 minutes after each meal. What should the nurse teach the patient to do? a. Increase the amount of fluid with meals. b. Eat foods that are higher in carbohydrates. c. Lie down for about 30 minutes after eating. d. Drink sugared fluids or eat candy after meals.

ANS: C The patient is experiencing symptoms of dumping syndrome, which may be reduced by lying down after eating. Increasing fluid intake and choosing high carbohydrate foods will increase the risk for dumping syndrome. Having a sweet drink or hard candy will correct the hypoglycemia that is associated with dumping syndrome but will not prevent dumping syndrome.

7. A young adult patient is admitted to the hospital for evaluation of right lower quadrant abdominal pain with nausea and vomiting. Which action should the nurse take? a. Assist the patient to cough and deep breathe. b. Palpate the abdomen for rebound tenderness. c. Suggest the patient lie on the side, flexing the right leg. d. Encourage the patient to sip clear, noncarbonated liquids.

ANS: C The patient's clinical manifestations are consistent with appendicitis. Lying still with the right leg flexed is often the most comfortable position. Checking for rebound tenderness frequently is unnecessary and uncomfortable for the patient. The patient should be NPO in case immediate surgery is needed. The patient will need to know how to cough and deep breathe postoperatively, but coughing will increase pain at this time.

48. A 72-yr-old male patient with dehydration caused by an exacerbation of ulcerative colitis is receiving 5% dextrose in normal saline at 125 mL/hour. Which assessment finding by the nurse is most important to report to the health care provider? a. Patient has not voided for the last 4 hours. b. Skin is dry with poor turgor on all extremities. c. Crackles are heard halfway up the posterior chest. d. Patient has had 5 loose stools over the previous 6 hours.

ANS: C The presence of crackles in an older patient receiving IV fluids at a high rate suggests volume overload and a need to reduce the rate of the IV infusion. The other data will also be reported but are consistent with the patient's age and diagnosis and do not require a change in the prescribed treatment.

44. After change-of-shift report, which patient should the nurse assess first? a. A 42-yr-old patient who has acute gastritis and ongoing epigastric pain b. A 70-yr-old patient with a hiatal hernia who experiences frequent heartburn c. A 60-yr-old patient with nausea and vomiting who is lethargic with dry mucosa d. A 53-yr-old patient who has dumping syndrome after a recent partial gastrectomy

ANS: C This patient is at high risk for problems such as aspiration, dehydration, and fluid and electrolyte disturbances. The other patients will also need to be assessed, but the information about them indicates symptoms that are typical for their diagnoses and are not life threatening.

41. Which care activity for a patient with a paralytic ileus is appropriate for the registered nurse (RN) to delegate to unlicensed assistive personnel (UAP)? a. Auscultation for bowel sounds b. Nasogastric (NG) tube irrigation c. Applying petroleum jelly to the lips d. Assessment of the nares for irritation

ANS: C UAP education and scope of practice include patient hygiene such as oral care. The other actions require education and scope of practice appropriate to the RN.

36. A patient is admitted to the emergency department with severe abdominal pain and rebound tenderness. Vital signs include temperature 102°F (38.3°C), pulse 120 beats/min, respirations 32 breaths/min, and blood pressure (BP) 82/54 mm Hg. Which prescribed intervention should the nurse implement first? a. Administer IV ketorolac 15 mg for pain relief. b. Draw a blood sample for a complete blood count (CBC). c. Infuse a liter of lactated Ringer's solution over 30 minutes. d. Send the patient for an abdominal computed tomography (CT) scan.

ANS: C The priority for this patient is to treat the patient's hypovolemic shock with fluid infusion. The other actions should be implemented after starting the fluid infusion.

19. The nurse is providing preoperative teaching for a patient scheduled for an abdominal-perineal resection. Which information will the nurse include? a. The patient will begin sitting in a chair at the bedside on the first postoperative day. b. IV antibiotics will be started at least 24 hours before surgery to reduce the bowel bacteria. c. An additional surgery in 8 to 12 weeks will be used to create an ileal-anal reservoir. d. The site where the stoma will be located will be marked on the abdomen preoperatively.

ANS: D A WOCN should select the site where the ostomy will be positioned and mark the abdomen preoperatively. The site should be within the rectus muscle, on a flat surface, and in a place that the patient is able to see. A permanent colostomy is created with this surgery. Sitting is contraindicated after an abdominal-perineal resection. Oral antibiotics (rather than IV antibiotics) are given to reduce colonic and rectal bacteria.

26. A 40-yr-old male patient has had a herniorrhaphy to repair an incarcerated inguinal hernia. Which patient teaching will the nurse provide before discharge? a. Soak in sitz baths several times each day. b. Cough 5 times each hour for the next 48 hours. c. Avoid use of acetaminophen (Tylenol) for pain. d. Apply a scrotal support and ice to reduce swelling.

ANS: D A scrotal support and ice are used to reduce edema and pain. Coughing will increase pressure on the incision. Sitz baths will not relieve pain and would not be of use after this surgery. Acetaminophen can be used for postoperative pain.

17. A patient with diverticulosis has a large bowel obstruction. The nurse will monitor for a. referred back pain. b. metabolic alkalosis. c. projectile vomiting. d. abdominal distention.

ANS: D Abdominal distention is seen in lower intestinal obstruction. Referred back pain is not a common clinical manifestation of intestinal obstruction. Metabolic alkalosis is common in high intestinal obstruction because of the loss of HCl acid from vomiting. Projectile vomiting is associated with higher intestinal obstruction.

27. Which breakfast choice indicates a patient's good understanding of information about a diet for celiac disease? a. Oatmeal with nonfat milk b. wheat toast with butter c. Bagel with low-fat cream cheese d. Corn tortilla with scrambled eggs

ANS: D Avoidance of gluten-containing foods is the only treatment for celiac disease. Corn does not contain gluten, but oatmeal and wheat do.

10. What should the nurse anticipate teaching a patient with a new report of heartburn? a. A barium swallow b. Radionuclide tests c. Endoscopy procedures d. Proton pump inhibitors

ANS: D Because diagnostic testing for heartburn that is probably caused by gastroesophageal reflux disease (GERD) is expensive and uncomfortable, proton pump inhibitors are frequently used for a short period as the first step in the diagnosis of GERD. The other tests may be used but are not usually the first step in diagnosis.

30. The nurse will plan to teach a patient with Crohn's disease who has megaloblastic anemia about the need for a. iron dextran infusions b. oral ferrous sulfate tablets. c. routine blood transfusions. d. cobalamin (B12) supplements.

ANS: D Crohn's disease frequently affects the ileum, where absorption of cobalamin occurs. Cobalamin must be administered regularly by nasal spray or IM to correct the anemia. Iron deficiency does not cause megaloblastic anemia. The patient may need occasional transfusions but not regularly scheduled transfusions.

3. Which menu choice best indicates that the patient is implementing the nurse's suggestion to choose high-calorie, high-protein foods? a. Baked fish with applesauce b. Beef noodle soup and canned corn c. Fresh fruit salad with yogurt topping d. Fried chicken with potatoes and gravy

ANS: D Foods that are high in calories include fried foods and those covered with sauces. High-protein foods include meat and dairy products. The other choices are lower in calories and protein.

42. An 80-yr-old patient who is hospitalized with peptic ulcer disease develops new-onset auditory hallucinations. Which prescribed medication will the nurse discuss with the health care provider before administration? a. Sucralfate (Carafate) b. Aluminum hydroxide c. Omeprazole (Prilosec) d. Metoclopramide (Reglan)

ANS: D Metoclopramide can cause central nervous system side effects ranging from anxiety to hallucinations. Hallucinations are not a side effect of proton pump inhibitors, mucosal protectants, or antacids.

27. A patient who takes a nonsteroidal antiinflammatory drug (NSAID) daily for the management of severe rheumatoid arthritis has recently developed melena. What should the nurse anticipate teaching the patient? a. Substitution of acetaminophen (Tylenol) for the NSAID b. Use of enteric-coated NSAIDs to reduce gastric irritation c. Reasons for using corticosteroids to treat the rheumatoid arthritis d. Misoprostol (Cytotec) to protect the gastrointestinal (GI) mucosa

ANS: D Misoprostol, a prostaglandin analog, reduces acid secretion and the incidence of upper GI bleeding associated with NSAID use. Enteric coating of NSAIDs does not reduce the risk for GI bleeding. Corticosteroids increase the risk for ulcer development and will not be substituted for NSAIDs for this patient. Acetaminophen will not be effective in treating rheumatoid arthritis.

33. Which question from the nurse would help determine if a patient's abdominal pain might indicate irritable bowel syndrome (IBS)? a. "Have you been passing a lot of gas?" b. "What foods affect your bowel patterns?" c. "Do you have any abdominal distention?" d. "How long have you had abdominal pain?"

ANS: D One criterion for the diagnosis of irritable bowel syndrome is the presence of abdominal discomfort or pain for at least 3 months. Abdominal distention, flatulence, and food intolerance are associated with IBS but are not diagnostic criteria.

28. The health care provider prescribes antacids and sucralfate (Carafate) for treatment of a patient's peptic ulcer. What should the nurse teach the patient to take? a. Sucralfate at bedtime and antacids before each meal b. Sucralfate and antacids together 30 minutes before meals c. Antacids 30 minutes before each dose of sucralfate is taken d. Antacids after meals and sucralfate 30 minutes before meals

ANS: D Sucralfate is most effective when the pH is low and should not be given with or soon after antacids. Antacids are most effective when taken after eating. Administration of sucralfate 30 minutes before eating and antacids just after eating will ensure that both drugs can be most effective. The other regimens will decrease the effectiveness of the medications.

38. Which assessment should the nurse perform first for a patient who just vomited bright red blood? a. Measuring the quantity of emesis b. Palpating the abdomen for distention c. Auscultating the chest for breath sounds d. Taking the blood pressure (BP) and pulse

ANS: D The nurse is concerned about blood loss and possible hypovolemic shock in a patient with acute gastrointestinal bleeding. BP and pulse are the best indicators of these complications. The other information is important to obtain, but BP and pulse rate are the best indicators for assessing intravascular volume.

10. A 60-yr-old man who is hospitalized with an abdominal wound infection has been eating very little and states, "Nothing on the menu sounds good." Which action by the nurse will be most effective in improving the patient's oral intake? a. Order six small meals daily. b. Make a referral to the dietitian. c. Teach the patient about high-calorie foods. d. Ask family members to bring favorite foods.

ANS: D The patient's statement that the hospital foods are unappealing indicates that favorite home-cooked foods might improve intake. The other interventions may also help improve the patient's intake, but the most effective action will be to offer the patient more appealing foods.

22. A patient is transferred from the recovery room to a surgical unit after a transverse colostomy. The nurse observes the stoma to be deep pink with edema and a small amount of sanguineous drainage. The nurse should a. place ice packs around the stoma. b. notify the surgeon about the stoma. c. monitor the stoma every 30 minutes. d. document stoma assessment findings.

ANS: D The stoma appearance indicates good circulation to the stoma. There is no indication that surgical intervention is needed or that frequent stoma monitoring is required. Swelling of the stoma is normal for 2 to 3 weeks after surgery, and an ice pack is not needed.

23) A client with peptic ulcer disease is undergoing pharmacologic therapy. After reviewing the medication administration record, the nurse suggests changing a prn pain order of ibuprofen to: 1. acetaminophen. 2. aspirin. 3. ketorolac. 4. naproxen.

Answer: 1 Explanation: Acetaminophen has analgesic and antipyretic qualities only. This drug should not worsen the progression of PUD. Page Ref: 1088

16) The nurse is caring for a client with irritable bowel syndrome (IBS). Which drugs would most likely be used to treat this disorder? 1. Antispasmodics 2. Saline cathartics 3. Sulfasalazine 4. Immunosuppressive agents

Answer: 1 Explanation: Antispasmodics may be used to treat irritable bowel syndrome. Page Ref: 1117

20) A client diagnosed with GERD asks the nurse what the treatment will be. The nurse correctly states: 1. "You likely will be instructed to change your lifestyle to see if that helps relieve your symptoms." 2. "We will start you on an over-the-counter agent because you have easy access to it and a small chance of overdosing on it." 3. "We need to attack this early, and will start with combination therapy." 4. "GERD will require a surgical treatment for symptom relief."

Answer: 1 Explanation: Before initiating pharmacotherapy, clients are usually advised to change lifestyle factors that might be contributing to the severity of GERD. Page Ref: 1087

3) A client who was taking large doses of ibuprofen (Motrin) for over a year developed a peptic ulcer. The client asks the nurse why this occurred. The nurse best answers the client by stating: 1. "Motrin works against your own body's ulcer defenses." 2. "It sounds like your stress might have caused your ulcer." 3. "Taking Motrin cannot cause a peptic ulcer." 4. "The natural defenses of your large intestine were overwhelmed."

Answer: 1 Explanation: Ibuprofen is a prostaglandin antagonist. This will decrease the defenses of the gastric mucosa and could precipitate a peptic ulcer. Page Ref: 1037

34) A nurse is ordered to administer lansoprazole to a client on a mechanical soft diet. How should the nurse administer this medication? 1. Sprinkle granules into applesauce, then administer. 2. Crush the tablet and stir it into pudding, then administer. 3. Ask the client to chew the medication. 4. First ask the client to drink an entire glass of water, then give the medication.

Answer: 1 Explanation: Lansoprazole granules may be sprinkled from the capsule into soft foods such as applesauce. Page Ref: 1092

10) During a pain assessment, a client reports stomach pain that feels like gnawing in the upper abdomen that occurs about 1-1/2 hours after eating meals. The nurse identifies these symptoms to be most consistent with: 1. a duodenal ulcer. 2. acute appendicitis. 3. an exacerbation of diverticulitis. 4. cholelithiasis.

Answer: 1 Explanation: The characteristic symptom of a duodenal ulcer is a gnawing or burning, upper abdominal pain that occurs 1 to 3 hours after a meal. The pain is worse when the stomach is empty and often disappears with ingestion of food. Page Ref: 1085

24) A client diagnosed with Barrett's esophagus might require the most efficient pharmacologic therapy of GERD. This would include the use of which class of drug? 1. Proton pump inhibitor 2. Antacid 3. Magnesium citrate 4. H2-receptor antagonist

Answer: 1 Explanation: The proton pump inhibitors reduce acid secretion to a greater extent than do the H2-receptor antagonists and have a longer duration of action. Page Ref: 1088

19) The nurse is caring for a client with PUD. What signs and symptoms does the nurse expect the client to exhibit? Select all that apply. 1. Anorexia 2. Weight loss 3. Vomiting 4. Heartburn 5. Belching

Answer: 1, 2, 3 Explanation: Anorexia is a symptom that is often associated with PUD. Weight loss is a symptom that is often associated with PUD. Vomiting is a symptom that is often associated with PUD and GERD. Page Ref: 1087

4) What are the principal physiologic stimuli that regulate acid secretion from the proton pump? Select all that apply. 1. Gastrin 2. Histamine (H2) 3. Acetylcholine 4. Intrinsic factor 5. Dopamine

Answer: 1, 2, 3 Explanation: Parietal cells receive messages from several sources, which tell them to increase or decrease acid production. These cells contain receptors for the hormone gastrin, which is a principal physiologic stimulus that regulates acid secretion from the proton pump. Parietal cells receive messages from several sources, which tell them to increase or decrease acid production. These cells contain receptors for histamine (H2), which is a principal physiologic stimulus that regulates acid secretion from the proton pump. Parietal cells receive messages from several sources, which tell them to increase or decrease acid production. These cells contain receptors for the neurotransmitter acetylcholine, which is a principal physiologic stimulus that regulates acid secretion from the proton pump. Page Ref: 1093

27) The nurse who is planning care for a client with GERD anticipates that the healthcare provider will order medications from which drug class? Select all that apply. 1. Proton pump inhibitors 2. H2-receptor antagonists 3. Antacids 4. Diuretics 5. Antianginals

Answer: 1, 2, 3 Explanation: Proton pump inhibitors are utilized in the treatment of GERD. H2-receptor antagonists are utilized in the treatment of GERD. Antacids are utilized in the treatment of GERD. Page Ref: 1088

22) The nurse caring for a client with PUD explains that the treatment goals include: Select all that apply. 1. immediate relief from symptoms. 2. promoting healing of the ulcer. 3. preventing complications of the disease. 4. preventing future recurrence of the disease. 5. reduction of the level of H. pylori by one third.

Answer: 1, 2, 3, 4 Explanation: The goals of PUD pharmacotherapy include providing immediate relief from symptoms. The goals of PUD pharmacotherapy include promoting healing of the ulcer. The goals of PUD pharmacotherapy include preventing complications of the disease. The goals of PUD pharmacotherapy include preventing future recurrence of the disease. Page Ref: 1088

8) The nurse is teaching the family of a client recently diagnosed with PUD. The nurse knows the family members understand the teaching when they state that the risk factors for the development of PUD include: Select all that apply. 1. infection with the bacterium Helicobacter pylori. 2. daily use of aspirin. 3. blood type AB. 4. consumption of caffeine-free soda. 5. smoking.

Answer: 1, 2, 5 Explanation: Helicobacter pylori is a risk factor for the development of PUD. Daily use of aspirin is a risk factor for the development of PUD. The use of tobacco products is a risk factor for the development of PUD. Page Ref: 1084

26) A client is being treated with sulfasalazine (Azulfidine) for inflammatory bowel disease (IBD). Which information is important for the nurse to provide to the client and family? Select all that apply. 1. It is important to avoid prolonged sun exposure, because the skin might be sensitive. 2. The drug should not be taken for longer than 12 weeks unless recommended by the healthcare provider. 3. Fluid intake should be 2 to 3 liters per day to avoid crystal formation in the urine. 4. The medication can color urine and skin orange-yellow. 5. The medication can affect female fertility.

Answer: 1, 3, 4 Explanation: Sulfasalazine can cause skin sensitivity to prolonged sun exposure. Fluid intake for a client taking sulfasalazine for IBD should be 2 to 3 liters per day to avoid crystal formation in the urine. Sulfasalazine can color urine and skin orange-yellow. Page Ref: 1115

31) The nurse is reviewing the medication administration record for a client who is receiving an initial regimen for the eradication of H. pylori. The nurse anticipates the inclusion of which medications for this client? Select all that apply. 1. Omeprazole 2. Metronidazole 3. Bismuth subsalicylate 4. Amoxicillin 5. Clarithromycin

Answer: 1, 4, 5 Explanation: Omeprazole is included in the initial medication regimen in the eradication of H. pylori. Amoxicillin is included in the initial medication regimen in the eradication of H. pylori. Clarithromycin is included in the initial medication regimen in the eradication of H. pylori. Page Ref: 1088

30) A client is prescribed omeprazole, bismuth subsalicylate, and metronidazole for the treatment of H. pylori. The nurse explains that combination therapy will accomplish which outcome? 1. Increase the number of ulcerations. 2. Eliminate the population of H. pylori. 3. Allow H. pylori to adhere to the gastric mucosa. 4. Immediately relieve symptoms of gastric ulceration.

Answer: 2 Explanation: A goal of combination therapy is to eliminate the population of H. pylori. Page Ref: 1088

17) A client complains of gastroesophageal symptoms every evening. The nurse would include what in the teaching to decrease the client's symptoms? 1. Moderating alcohol use 2. Avoiding acidic foods 3. Sleeping with the legs elevated on pillows 4. Eating smaller meals at least 1 hour before sleep

Answer: 2 Explanation: Acidic foods should be avoided in clients with GERD. Page Ref: 1086

37) A client in a long-term care facility is ordered to receive ranitidine (Zantac). When should the nurse administer this medication? 1. At bedtime with water 2. After the evening meal 3. Before eating in the morning 4. One hour before meals

Answer: 2 Explanation: The ideal time to administer ranitidine is after the evening meal, because the buffering effect of food can increase the therapeutic effect. Page Ref: 1093

15) A client with a chronic cough, wheezing, and a sore throat states these symptoms have been present for 3 months, and asthma is not the cause. What does the nurse suspect to be causing this client's respiratory issues and sore throat? 1. Zollinger-Ellison syndrome 2. Gastroesophageal reflex disease 3. Crohn's disease 4. A gastric ulcer

Answer: 2 Explanation: There is growing evidence that clients with GERD also can present with symptoms such as chronic cough, wheezing, bronchitis, sore throat, or hoarseness. Page Ref: 1086

29) Effective client teaching regarding the treatment of H. pylori has been done for a client when the client states: 1. "I will stop taking this medication when my symptoms subside." 2. "I will be sure to take all of the medication exactly as prescribed." 3. "I will stop taking this medication if I get an upset stomach." 4. "I will take this medication when I experience pain or heartburn."

Answer: 2 Explanation: This statement indicates that effective client teaching has been performed. Page Ref: 1088

2) A client with a diagnosis of peptic ulcer disease is prescribed omeprazole (Prilosec) and asks the nurse where in the body this medication will work. The nurse correctly answers: 1. "On the gastric mucosa layer." 2. "On the mucosa of the trachea." 3. "On the surface of parietal cells." 4. "On the H2 receptors."

Answer: 3 Explanation: Omeprazole is a proton pump inhibitor and works on the proton pump, which is located on the surface of parietal cells. Page Ref: 1091

5) What client has the greatest risk of having a peptic ulcer? 1. A 24-year-old who complains of being stressed mentally 2. A 52-year-old who smokes 3. A 56-year-old who is taking prednisone and ibuprofen for chronic low-back pain 4. A 48-year-old who is taking acetaminophen and aspirin to reduce fever

Answer: 3 Explanation: This client has two risk factors for developing a peptic ulcer, making this the best choice. Page Ref: 1084

17) A client is being treated for inflammatory bowel disease (IBD). The nurse anticipates receiving orders for which medications? Select all that apply. 1. Loperamide (Imodium) 2. Doxepin (Sinequan) 3. Sulfasalazine (Azulfidine) 4. Prednisone 5. Azathioprine (Imuran)

Answer: 3, 4, 5 Explanation: The first step of IBD treatment is usually with a 5-aminosalicylic acid (5-ASA) agent, which can include sulfasalazine (Azulfidine). When IBD is especially severe or when clients have not responded well to the 5-ASA drugs, oral corticosteroids such as prednisone are used. Should therapy with corticosteroids fail, or if they are needed for prolonged periods, step 3 of IBD therapy includes immunosuppressive agents, such as azathioprine (Imuran). Page Ref: 1112

1) The nurse correctly states that which function is not characteristic of the parietal cells? 1. Secretion of intrinsic factor 2. Microbial control 3. Secretion of hydrochloric acid 4. Secretion of pepsinogen

Answer: 4 Explanation: Chief cells, not parietal cells, secrete the enzyme pepsinogen. Page Ref: 1083

32) The nurse is performing a head-to-toe assessment on a client taking aluminum hydroxide (AlternaGEL). Why must the nurse closely assess for bowel changes? 1. Hyperactive bowel sounds indicate imminent vomiting, suction should be made available. 2. Hypoactive bowel sounds indicate duodenal ulcerations. 3. Hematochezia is a sign that the medication is working. 4. A distended abdomen could indicate constipation, a side effect of this medication.

Answer: 4 Explanation: During treatment with aluminum hydroxide, the nurse must assess for bowel changes. Magnesium-based products can cause diarrhea, and those with calcium and aluminum can cause constipation. Page Ref: 1096

7) The healthcare team is attempting to determine cause for a client's duodenal ulcer. Which factors are not associated with peptic ulcer disease (PUD)? 1. Smoking cigarettes 2. Having a parent diagnosed with a peptic ulcer 3. Excessive stress at home 4. Having type A blood

Answer: 4 Explanation: Having type O blood, not type A, places one at risk for developing a peptic ulcer. Page Ref: 1084

38) A client brings her mother to the office and states, "I'm worried about Mom, she takes aluminum hydroxide up to three times a day for this pain in her abdomen." Which statement about aluminum hydroxide is true? 1. Aluminum hydroxide can cause acute appendicitis and should be stopped immediately. 2. A transient abdominal discomfort occurs with the use of aluminum hydroxide and there is no need for concern. 3. It is safe to take aluminum hydroxide up to three times daily, due to its safety profile. 4. Antacids are for occasional use only, further investigation is warranted.

Answer: 4 Explanation: OTC medications taken long term can mask symptoms of more serious diseases such as cancer or complicated ulcers. This client's discomfort should be further investigated. Page Ref: 1094

33) A client is prescribed ranitidine (Zantac). The nurse checks the client's BUN and serum creatinine levels prior to administering the drug for the first time. The rationale for checking these labs is: 1. serum BUN and creatinine should be checked on all clients. 2. that the drug is primarily biometabolized by the liver. 3. to identify a hidden drug allergy. 4. that the drug is primarily excreted by the kidneys.

Answer: 4 Explanation: These drugs are mainly excreted by the kidneys, and clients with diminished kidney function require smaller doses. Page Ref: 1093

The nurse determines that the goals of dietary teaching have been met when the patient with celiac disease selects from the menu a. scrambled eggs and sausage. b. buckwheat pancakes with syrup. c. oatmeal, skim milk, and orange juice. d. yogurt, strawberries, and rye toast with butter.

Correct answer: a Rationale: Celiac disease is treated with lifelong avoidance of dietary gluten (wheat, barley, oats, rye products). Although pure oats do not contain gluten, oat products can become contaminated with wheat, rye, and barley during the milling process. Gluten is also found in some medications and in many food additives, preservatives, and stabilizers.

A patient has an elevated blood level of indirect (unconjugated) bilirubin. One cause of this finding is that a. the gallbladder is unable to contract to release stored bile. b. bilirubin is not being conjugated and excreted into the bile by the liver. c. the Kupffer cells in the liver are unable to remove bilirubin from the blood. d. there is an obstruction in the biliary tract preventing flow of bile into the small intestine.

Correct answer: b Rationale: Bilirubin is a pigment derived from the breakdown of hemoglobin and is insoluble in water. Bilirubin is bound to albumin for transport to the liver and is referred to as unconjugated. An indirect bilirubin determination is a measurement of unconjugated bilirubin, and the level may be elevated in hepatocellular and hemolytic conditions.

In contrast to diverticulitis, the patient with diverticulosis a. has rectal bleeding. b. often has no symptoms. c. has localized cramping pain. d. frequently develops peritonitis.

Correct answer: b Rationale: Many people with diverticulosis have no symptoms. Patients with diverticulitis have symptoms of inflammation. Diverticulitis can lead to obstruction or perforation.

An 80-year-old man states that, although he adds a lot of salt to his food, it still does not have much taste. The nurse's response is based on the knowledge that the older adult a. should not experience changes in taste. b. has a loss of taste buds, especially for sweet and salt. c. has some loss of taste but no difficulty chewing food. d. loses the sense of taste because the ability to smell is decreased.

Correct answer: b Rationale: Older adults have decreased numbers of taste buds and a decreased sense of smell. These age-related changes diminish the sense of taste (especially of salty and sweet substances).

As gastric contents move into the small intestine, the bowel is normally protected from the acidity of gastric contents by the a. inhibition of secretin release. b. release of bicarbonate by the pancreas. c. release of pancreatic digestive enzymes. d. release of gastrin by the duodenal mucosa

Correct answer: b Rationale: The hormone secretin stimulates the pancreas to secrete fluid with a high concentration of bicarbonate. This alkaline secretion enters the duodenum and neutralizes acid in the chyme.

In planning care for the patient with Crohn's disease, the nurse recognizes that a major difference between ulcerative colitis and Crohn's disease is that Crohn's disease a. frequently results in toxic megacolon. b. causes fewer nutritional deficiencies than ulcerative colitis. c. often recurs after surgery, whereas ulcerative colitis is curable with a colectomy d. is manifested by rectal bleeding and anemia more often than is ulcerative colitis.

Correct answer: c Rationale: Ulcerative colitis affects only the colon and rectum; it can cause megacolon and rectal bleeding, but not nutrient malabsorption. Surgical removal of the colon and rectum cures it. Crohn's disease usually involves the ileum, where bile salts and vitamin cobalamin are absorbed. After surgical treatment, disease recurrence at the site is common.

When the nurse is assessing the health perception-health maintenance pattern as related to GI function, an appropriate question to ask is a. "What is your usual bowel elimination pattern?" b. "What percentage of your income is spent on food?" c. "Have you traveled to a foreign country in the last year?" d. "Do you have diarrhea when you are under a lot of stress?"

Correct answer: c Rationale: When assessing gastrointestinal function in relation to the health perception-health management pattern, the nurse should ask the patient about recent foreign travel with possible exposure to hepatitis, parasitic infestation, or bacterial infection.

What should a patient be taught after a hemorrhoidectomy? a. Take mineral oil before bedtime. b. Eat a low-fiber diet to rest the colon. c. Administer oil-retention enema to empty the colon. d. Use prescribed pain medication before a bowel movement.

Correct answer: d Rationale: After a hemorrhoidectomy, the patient usually dreads the first bowel movement and often resists the urge to defecate. Give pain medication before the bowel movement to reduce discomfort. The patient should avoid constipation and straining. A high-fiber diet can reduce constipation. A stool softener such as docusate (Colace) is usually ordered for the first few postoperative days. If the patient does not have a bowel movement within 2 to 3 days, an oil-retention enema is administered.

The nurse is teaching the patient and family that peptic ulcers are a. caused by a stressful lifestyle and other acid-producing factors such as H. pylori. b. inherited within families and reinforced by bacterial spread of Staphylococcus aureus in childhood. c. promoted by factors that tend to cause oversecretion of acid, such as excess dietary fats, smoking and H. pylori. d. promoted by a combination of factors that may result in erosion of the gastric mucosa, including certain drugs and alcohol.

Correct answer: d Rationale: Peptic ulcers develop only in the presence of an acidic environment. However, an excess of hydrochloric acid (HCl) may not be necessary for ulcer development. The back diffusion of HCl into the gastric mucosa results in cellular destruction and inflammation. Histamine is released from the damaged mucosa, which results in vasodilation and increased capillary permeability and further secretion of acid and pepsin. A variety of agents (certain infections, medications, and lifestyle factors) can damage the mucosal barrier. Helicobacter pylori can alter gastric secretion and produce tissue damage, which leads to peptic ulcer disease. Ulcerogenic drugs, such as aspirin and NSAIDs, inhibit synthesis of prostaglandins, increase gastric acid secretion, and reduce the integrity of the mucosal barrier. Patients taking corticosteroids, anticoagulants, and selective serotonin reuptake inhibitors are at increased risk for ulcers. High alcohol intake stimulates acid secretion and is associated with acute mucosal lesions. Coffee (caffeinated and uncaffeinated) is a strong stimulant of gastric acid secretion.

The nurse performs a detailed assessment of the abdomen of a patent with a possible bowel obstruction, knowing that manifestations of an obstruction in the large intestine are (select all that apply) a. persistent abdominal pain. b. marked abdominal distention. c. diarrhea that is loose or liquid. d. colicky, severe, intermittent pain. e. profuse vomiting that relieves abdominal pain.

Correct answers: a, b Rationale: With lower intestinal obstructions, abdominal distention is markedly increased and pain is persistent. Onset of a large intestine obstruction is gradual, vomiting is rare, and there is usually absolute constipation, not diarrhea.

Assessment findings suggestive of peritonitis include (select all that apply) a. rebound tenderness. b. a soft, distended abdomen. c. dull, intermittent abdominal pain d. shallow respirations with bradypnea e. observing that the patient is lying still.

Correct answers: a, e Rationale: With peritoneal irritation, the abdomen is hard, and the patient has severe continuous abdominal pain that is worse with any sudden movement. Palpating the abdomen and releasing the hands suddenly causes sudden movement within the abdomen and severe pain. This is called rebound tenderness. The patient lies very still and takes shallow breaths. Abdominal distention, tachypnea, fever, and tachycardia may occur.

The teaching plan for the patient being discharged after an acute episode of upper GI bleeding includes information concerning the importance of (select all that apply) a. limiting alcohol intake to one serving per day. b. only taking aspirin with milk or bread products. c. avoiding taking aspirin and drugs containing aspirin. d. only taking only drugs prescribed by the health care provider e. taking all drugs 1 hour before mealtime to prevent further bleeding

Correct answers: c, d Rationale: Before discharge, the patient with upper gastrointestinal (GI) bleeding and the caregiver should be taught how to avoid future bleeding episodes. Ulcer disease and drug or alcohol abuse can cause upper GI bleeding. Help make the patient and caregiver aware of the consequences of noncompliance with drug therapy. Emphasize that no drugs (especially aspirin and nonsteroidal antiinflammatory drugs [NSAIDs]) other than those prescribed by the HCP should be taken. Alcohol should be eliminated because it is a source of irritation and interfere with tissue repair.

The nurse is assessing a 50-year-old woman admitted with a possible bowel obstruction. Which assessment finding would be expected in this patient? Tympany to abdominal percussion Aortic pulsation visible in epigastric region High-pitched sounds on abdominal auscultation Liver border palpable 1 cm below the right costal margin

High-pitched sounds on abdominal auscultation The bowel sounds are more high pitched (rushes and tinkling) when the intestines are under tension, as in intestinal obstruction. Bowel sounds may also be diminished or absent with an intestinal obstruction. Normal findings include aortic pulsations on inspection and tympany with percussion, and the liver may be palpable 1 to 2 cm along the right costal margin.

Duodenal and gastric ulcers have similar as well as differentiating features. What characteristics are unique to duodenal ulcers (select all that apply)? a. Pain is relieved with eating food. b. They have a high recurrence rate. c. Increased gastric acid secretion occurs. d. Associated with Helicobacter pylori infection. e. Hemorrhage, perforation, and obstruction may result. f. There is burning and cramping in the midepigastric area.

a, c, f. Duodenal ulcers have increased HCI gastric secretion, which causes the burning and cramping in the midepigastric area; the pain is relieved with food. The other options occur with both duodenal and gastric ulcers.

The patient calls the clinic and describes a bump at the site of a previous incision that disappears when he lies down. The nurse suspects that this is which type of hernia (select all that apply)? a. Ventral b. Inguinal c. Femoral d. Reducible e. Incarcerated f. Strangulated

a, d. The ventral or incisional hernia is caused by a weakness of the abdominal wall at the site of a previous incision. It is reducible because it returns to the abdominal cavity. Inguinal hernias are at the weak area of the abdominal wall, where the spermatic cord in men or the round ligament in women emerges. A femoral hernia is a protrusion through the femoral ring into the femoral canal. Incarcerated hernias do not reduce. Strangulation occurs when the blood supply to an irreducible hernia is compromised.

The nurse is preparing to administer famotidine to a postoperative patient with a colostomy. The patient states they do not have heartburn. What response by the nurse would be the most appropriate? a. "It will reduce the amount of acid in the stomach." b. "It will prevent air from accumulating in the stomach, causing gas pains." c. "It will prevent the heartburn that occurs as a side effect of general anesthesia." d. "The stress of surgery is likely to cause stomach bleeding if you do not receive it."

a. "It will reduce the amount of acid in the stomach." Famotidine is an H2-receptor antagonist that inhibits gastric HCl secretion and thus minimizes damage to gastric mucosa while the patient is not eating a regular diet after surgery. Famotidine does not prevent air from accumulating in the stomach or stop the stomach from bleeding. Heartburn is not a side effect of general anesthesia.

The results of a patient's recent endoscopy indicate the presence of peptic ulcer disease (PUD). Which teaching point should the nurse provide to the patient based on this new diagnosis? a. "It would be beneficial for you to eliminate drinking alcohol." b. "You'll need to drink at least two to three glasses of milk daily." c. "Many people find that a minced or pureed diet eases their symptoms of PUD." d. "Taking medication will allow you to keep your present diet while minimizing symptoms."

a. "It would be beneficial for you to eliminate drinking alcohol." Alcohol increases the amount of stomach acid produced. so it should be avoided. Although there is no specific recommended dietary modification for PUD, most patients find it necessary to make some sort of dietary modifications to minimize symptoms. Milk may exacerbate PUD.

The stable patient has a gastrostomy tube for enteral feeding. Which care could the RN delegate to the LPN? Select all that apply. a. Administer bolus or continuous feedings. b. Evaluate the nutritional status of the patient. c. Administer medications through the gastrostomy tube. d. Monitor for complications related to the tube and enteral feeding. e. Teach the caregiver about feeding via the gastrostomy tube at home.

a. Administer bolus or continuous feedings. c. Administer medications through the gastrostomy tube. For the stable patient, the LPN can administer bolus or continuous feedings and administer medications through the gastrostomy. The RN must evaluate the nutritional status of the patient, monitor for complications related to the tube and the enteral feeding, and teach the caregiver about feeding via the gastrostomy tube at home.

Priority Decision: An 82-year-old man is admitted with an acute attack of diverticulitis. What is most important for the nurse to include in his care? a. Monitor for signs of peritonitis. b. Treat with daily medicated enemas. c. Prepare for surgery to resect the involved colon. d. Provide a heating pad to apply to the left lower quadrant.

a. Diverticulitis can erode the bowel wall and perforate into the peritoneum. Abscesses may form to wall off the area of perforation, but complete perforation with peritonitis may occur. Systemic antibiotic therapy is often used, but medicated enemas would increase intestinal motility and increase the possibility of perforation, as would the application of heat. Surgery is only necessary to drain abscesses or to resect an obstructing inflammatory mass.

The nurse plans teaching for the patient with a colostomy, but the patient refuses to look at the nurse or the stoma stating, "I just can't see myself with this thing." What is the best nursing intervention for this patient? a. Encourage the patient to share concerns and ask questions. b. Refer the patient to a chaplain to help cope with this situation. c. Explain that there is nothing the patient can do about it and must take care of it. d. Tell the patient that learning about it will prevent stool leaking and the sounds of flatus.

a. Encouraging the patient to share concerns and ask questions will help the patient begin to adapt to living with the colostomy. The other options do not support the patient and do not portray the nurse's focus on helping the patient or treating the patient as an individual.

The nurse is preparing a patient for a capsule endoscopy. What should the nurse ensure is included in the preparation? a. Ensure the patient understands the required bowel preparation. b. Have the patient return to the procedure room for removal of the capsule. c. Teach the patient to maintain a clear liquid diet throughout the procedure. d. Explain to the patient that conscious sedation will be used during capsule placement.

a. Ensure the patient understands the required bowel preparation. A capsule endoscopy study involves the patient performing a bowel prep to cleanse the bowel before swallowing the capsule. The patient will be on a clear liquid diet for 1 to 2 days before the procedure and will remain NPO for 4 to 6 hours after swallowing the capsule. The capsule is disposable and will pass naturally with the bowel movement, although the monitoring device will need to be removed.

The patient has parenteral nutrition (PN) infusing with amino acids and dextrose. During shift change, the nurse reports the tubing, bag, and dressing were changed 20 hours ago. What care should the incoming nurse plan to deliver? Select all that apply. a. Giving the patient insulin if needed b. Ensuring that the next bag has been ordered c. Checking amount of solution left in the bag d. Assessing the insertion site and change the tubing e. Verifying the accuracy of the new solution and ingredients

a. Giving the patient insulin if needed b. Ensuring that the next bag has been ordered c. Checking amount of solution left in the bag e. Verifying the accuracy of the new solution and ingredients The nurse should identify the amount of PN left in the bag when initiating care and request more if needed. Abrupt withdrawal of PN can cause hypoglycemia. The nurse should anticipate pharmacy preparation of a new bag may take significant time especially if additives are ordered. PN solutions are changed every 24 hours. The label on the bag should be verified with the order to ensure accuracy. The patient would receive insulin if hyperglycemic related to dextrose content parenteral nutrition or underlying diabetes mellitus. Sliding-scale coverage or addition of regular insulin to the parenteral nutrition would be provided if ordered. The insertion site should be monitored, but the tubing is only changed every 72 hours unless lipids are being used.

Priority Decision: A patient with a gastric outlet obstruction has been treated with NG decompression. After the first 24 hours, the patient develops nausea and increased upper abdominal bowel sounds. What is the priority action by the nurse? a. Check the patency of the NG tube. b. Place the patient in a recumbent position. c. Assess the patient's vital signs and circulatory status. d. Encourage the patient to deep breathe and consciously relax.

a. If symptoms of gastric outlet obstruction, such as nausea, vomiting, and stomach distention, occur while the patient is NPO status or has an NG tube, the patency with the of the NG tube should first be assessed. A recumbent position should not be used in a patient with a gastric outlet obstruction because it increases abdominal pressure on the stomach. Vital signs and circulatory status assessment are important if hemorrhage or preparation is suspected. Deep breathing and relaxation may help some patients with nausea, but not when stomach contents are obstructed from flowing into the intestine.

Two days after a colectomy for an abdominal mass, a patient reports gas pains and abdominal distention. The nurse plans care for the patient based on the knowledge that the symptoms are occurring as a result what event? a. Impaired peristalsis b. Irritation of the bowel c. Nasogastric suctioning d. Inflammation of the incision site

a. Impaired peristalsis Until peristalsis returns to normal after anesthesia, the patient may experience slowed gastrointestinal motility, leading to gas pains and abdominal distention. Irritation of the bowel, nasogastric suctioning, and inflammation of the surgical site do not cause gas pains or abdominal distention.

Priority Decision: A patient with ulcerative colitis has a total proctocolectomy with formation of a terminal ileum stoma. What is the most important nursing intervention for this patient postoperatively? a. Measure the ileostomy output to determine the status of the patient's fluid balance. b. Change the ileostomy appliance every 3 to 4 hours to prevent leakage of drainage onto the skin. c. Emphasize that the ostomy is temporary and the ileum will be reconnected when the large bowel heals. d. Teach the patient about the high-fiber, low-carbohydrate diet required to maintain normal ileostomy drainage.

a. Initial output from a newly formed ileostomy may be as high as 1500 to 2000mL daily, and intake and output must be accurately monitored for fluid and electrolyte imbalance. Ileostomy bags may have to be emptied every 3 to 4 hours, but the appliance should not be changed for several days unless there is leakage onto the skin. A terminal ileum stoma is permanent, and the entire colon has been removed. A return to a normal, presurgical diet is the goal for the patient with an ileostomy, with restrictions based only on the patient's individual tolerances.

A hospitalized patient has just been diagnosed with diarrhea due to Clostridium difficile. Which nursing interventions should be included in the patient's plan of care? Select all that apply. a. Initiate contact isolation precautions. b. Place the patient on a clear liquid diet. c. Disinfect the room with 10% bleach solution. d. Teach any visitors to wear gloves and gowns. e. Use hand sanitizer before and after patient or bodily fluid contact.

a. Initiate contact isolation precautions. c. Disinfect the room with 10% bleach solution. d. Teach any visitors to wear gloves and gowns. Initiation of contact isolation precautions must be done immediately with a patient with C. difficile, which includes washing hands with soap and water before and after patient or bodily fluid contact. Alcohol-based sanitizers are ineffective. Visitors need to be taught to wear gloves and gowns and wash hands. A clear liquid diet is not necessary. The room will be disinfected with 10% bleach solution when the patient is dismissed and may be done periodically during the patient's stay, depending on the agency policy.

What type of bleeding will a patient with peptic ulcer disease with a slow upper GI source of bleeding have? a. Melena b. Occult blood c. Coffee-ground emesis d. Profuse bright-red hematemesis

a. Melena is black, tarry stools from slow bleeding from an upper gastrointestinal (GI) source when blood passes through the GI tract and is digested. Occult blood is the presence of guaiac-positive stools or gastric aspirate. Coffee-ground emesis is blood that has been in the stomach for some time and has reacted with gastric secretions. Profuse bright-red hematemesis is arterial blood that has not been in contact with gastric secretions, as in esophageal or oral bleeding.

A patient with a history of peptic ulcer disease has presented to the emergency department with severe abdominal pain and a rigid, boardlike abdomen. The health care provider suspects a perforated ulcer. Which interventions should the nurse anticipate? a. Providing IV fluids and inserting a nasogastric (NG) tube b. Administering oral bicarbonate and testing the patient's gastric pH level c. Performing a fecal occult blood test and administering IV calcium gluconate d. Starting parenteral nutrition and placing the patient in a high-Fowler's position

a. Providing IV fluids and inserting a nasogastric (NG) A perforated peptic ulcer requires IV replacement of fluid losses and continued gastric aspiration by NG tube. Nothing is given by mouth, and gastric pH testing is not a priority. Calcium gluconate is not a medication directly relevant to the patient's suspected diagnosis, and parenteral nutrition is not a priority in the short term.

A patient with inflammatory bowel disease has a nursing diagnosis of impaired nutritional status; etiology: decreased nutritional intake and decreased intestinal absorption. Which assessment data support this nursing diagnosis? a. Pallor and hair loss b. Frequent diarrhea stools c. Anorectal excoriation and pain d. Hypotension and urine output below 30 mL/hr

a. Signs of malnutrition include pallor from anemia, hair loss, bleeding, cracked gingivae, and muscle weakness, which support a nursing diagnosis that identifies impaired nutrition. Diarrhea may contribute to malnutrition but is not a defining characteristic for this patient. Anorectal excoriation and pain relate to problems with skin integrity. Hypotension relates to problems with fluid deficit.

Which patient is at highest risk of having a gastric ulcer? a. 55-year-old female smoker with nausea and vomiting b. 45-year-old female admitted for illicit drug detoxification c. 27-year-old male who is being divorced and has back pain d. 37-year-old male smoker who was in an accident while looking for a job

a. The 55-year-old female smoker experiencing nausea and vomiting is more likely to have a gastric ulcer. The other patients are not in the highest-risk age range or do not have enough risk factors. Although lower socioeconomic status, smoking, and drug use increase the risk of gastric ulcers, these patients are more likely to have duodenal ulcers but further assessment is needed.

The wound, ostomy, and continence nurse (WOCN) selects the site where the ostomy will be placed. What should be included in site consideration? a. The patient must be able to see the site. b. The site should be outside the rectus muscle area. c. It is easier to seal the drainage bag to a protruding area. d. A waistline site will allow using a belt to hold the appliance in place.

a. The patient must be able to see the site. In selection of the ostomy site, the WOC nurse will want a site visible to the patient so the patient can take care of it, within the rectus muscle to avoid hernias, and on a flat surface to more easily create a good seal with the drainage bag. Care should be taken to avoid skin creases, scars, and belt lines, which can interfere with the adherence of the appliance.

Regardless of the precipitating factor, what causes the injury to mucosal cells in peptic ulcers? a. Acid back diffusion into the mucosa b. The release of histamine from gastrointestinal (GI) cells c. Ammonia formation in the mucosal wall d. Breakdown of the gastric mucosal barrier

a. The ultimate damage to the tissues of the stomach and duodenum, precipitating ulceration, is acid back diffusion into the mucosa. The gastric mucosal barrier is protective of the mucosa but without the acid environment and damage, ulceration does not occur. Ammonia formation by H. pylori and release of histamine impair the barrier but are not directly responsible for tissue injury.

A patient complaining of nausea receives a dose of metoclopramide. Which potential adverse effect should the nurse tell the patient to report? a. Tremors b. Constipation c. Double vision d. Numbness in fingers and toes

a. Tremors Extrapyramidal side effects, including tremors and tardive dyskinesias, may occur with metoclopramide administration. Constipation, double vision, and numbness in fingers and toes are not adverse effects of metoclopramide.

A patient is receiving peripheral parenteral nutrition. The parenteral nutrition solution is completed before the new solution arrives on the unit. The nurse gives a. 20% intralipids. b. 5% dextrose solution. c. 0.45% normal saline solution. d. 5% lactated Ringer's solution.

b. 5% dextrose solution.

On examining a patient 8 hours after having surgery to create a colostomy, what should the nurse expect to find? a. Hyperactive, high-pitched bowel sounds b. A brick-red, puffy stoma that oozes blood c. A purplish stoma, shiny and moist with mucus d. A small amount of liquid fecal drainage from the stoma

b. A normal new colostomy stoma should appear rose to brick-red, have mild to moderate edema, and have a small amount of bleeding or oozing of blood when touched. A purplish stoma indicates inadequate blood supply and should be reported. Bowel sounds after extensive bowel surgery will be diminished or absent. The colostomy will not have any fecal drainage for 2 to 4 days, but there may be some earlier mucus or serosanguineous drainage.

A colectomy is scheduled for a patient with ulcerative colitis. The nurse should plan to include which prescribed measure in the preoperative preparation of this patient? a. Instruction on irrigating a colostomy b. Administration of a cleansing enema c. A high-fiber diet the day before surgery d. Administration of IV antibiotics for bowel preparation

b. Administration of a cleansing enema Preoperative preparation for bowel surgery typically includes bowel cleansing with antibiotics, such as oral neomycin and cleansing enemas, including Fleet enemas. Instructions to irrigate the colostomy will be done postoperatively. Oral antibiotics are given preoperatively and an IV antibiotic may be used in the operating room. A clear liquid diet will be used the day before surgery with the bowel cleansing.

What should the nurse emphasize when teaching patients at risk for upper GI bleeding to prevent bleeding episodes? a. All stools and vomitus must be tested for the presence of blood. b. The use of over-the-counter (OTC) medications of any kind should be avoided. c. Antacids should be taken with all prescribed medications to prevent gastric irritation. d. Misoprostol (Cytotec) should be used to protect the gastric mucosa in individuals with peptic ulcers.

b. All OTC drugs should be avoided because their contents may include drugs that are contraindicated because of the irritating effects on the gastric mucosa. Patients are taught to test suspicious vomitus or stools for occult blood, but all stools do not have to be tested. Antacids cannot be taken with all medications because they prevent the absorption of many drugs. Patients with a history of ulcers who must take low-dose aspirin are prescribed misoprostol to protect the gastric mucosa.

The nurse is caring for a postoperative patient who has just vomited yellow green liquid and reports nausea. Which action would be an appropriate nursing intervention? a. Offer the patient an herbal supplement such as ginseng. b. Apply a cool washcloth to the forehead and provide mouth care. c. Take the patient for a walk in the hallway to promote peristalsis. d. Discontinue any medications that may cause nausea or vomiting.

b. Apply a cool washcloth to the forehead and provide mouth care. Cleansing the face and hands with a cool washcloth and providing mouth care are appropriate comfort interventions for nausea and vomiting. Ginseng is not used to treat postoperative nausea and vomiting. Unnecessary activity should be avoided. The patient should rest in a quiet environment. Medications may be temporarily withheld until the acute phase is over, but the medications should not be discontinued without consultation with the health care provider.

Priority Decision: When caring for a patient with irritable bowel syndrome (IBS), what is most important for the nurse to do? a. Recognize that IBS is a psychogenic illness that cannot be definitively diagnosed. b. Develop a trusting relationship with the patient to provide support and symptomatic care. c. Teach the patient that a diet high in fiber will relieve the symptoms of both diarrhea and constipation. d. Inform the patient that new medications are available and effective for treatment of IBS manifested by either diarrhea or constipation.

b. Because there is no definitive treatment for irritable bowel syndrome (IBS), patients become frustrated and discouraged with uncontrolled symptoms. It is important to develop a trusting relationship that will support the patient as different treatments are implemented and evaluated. Although IBS can be precipitated and aggravated by stress and emotions, it is not a psychogenic illness. High-fiber diets may help, but they may also increase the bloating and gas pains of IBS. Medications are available, but use is individualized because of side effects.

A patient who has sustained severe burns in a motor vehicle accident is starting parenteral nutrition (PN). Which principle should guide the nurse's administration of PN? a. Administration of PN requires clean technique. b. Central PN requires rapid dilution in a large volume of blood. c. Peripheral PN delivery is preferred over the use of a central line. d. Only water-soluble medications may be added to the PN by the nurse.

b. Central PN requires rapid dilution in a large volume of blood. Central PN is hypertonic and requires rapid dilution in a large volume of blood. Because PN is an excellent medium for microbial growth, aseptic technique is necessary during administration. Administration through a central line is preferred over the use of peripheral PN, and the nurse may not add any medications to PN.

The nurse should recognize that the liver performs which functions? Select all that apply. a. Bile storage b. Detoxification c. Protein metabolism d. Steroid metabolism e. Red blood cell (RBC) destruction

b. Detoxification c. Protein metabolism d. Steroid metabolism e. Red blood cell (RBC) destruction The liver performs multiple major functions that aid in the maintenance of homeostasis. These include metabolism of proteins and steroids as well as detoxification of drugs and metabolic waste products. The Kupffer cells of the liver participate in the breakdown of old RBCs. The liver produces bile, but storage occurs in the gallbladder.

The nurse is caring for a patient treated with IV fluid therapy for severe vomiting. As the patient recovers and begins to tolerate oral intake, which food choice would be most appropriate? a. Iced tea b. Dry toast c. Hot coffee d. Plain yogurt

b. Dry toast Dry toast or crackers may alleviate the feeling of nausea and prevent further vomiting. Water is the initial fluid of choice. Extremely hot or cold liquids and fatty foods are generally not well tolerated.

Which focused assessments would have priority in the care of a patient recently started on parenteral nutrition (PN)? a. Skin integrity and skin turgor b. Electrolyte levels and daily weights c. Auscultation of lung and bowel sounds d. Peripheral edema and level of consciousness

b. Electrolyte levels and daily weights The use of PN necessitates frequent and thorough assessments. Key assessments include daily weights and close monitoring of electrolyte levels. Assessments of bowel sounds, integument, peripheral edema, level of consciousness, and lung sounds, may be variously performed, but close monitoring of fluid and electrolyte balance supersedes these in importance related to the PN.

A patient is seeking emergency care after choking on a piece of steak. The nursing assessment reveals a history of alcoholism, cigarette smoking, and hemoptysis. Which diagnostic study is most likely to be performed on this patient? a. Barium swallow b. Endoscopic biopsy c. Capsule endoscopy d. Endoscopic ultrasonography

b. Endoscopic biopsy Because of this patient's history of excessive alcohol intake, smoking, and hemoptysis and the current choking episode, cancer may be present. A biopsy is necessary to make a definitive diagnosis of carcinoma, so an endoscope will be used to obtain a biopsy and observe other abnormalities as well. A barium swallow may show narrowing of the esophagus, but it is more diagnostic for achalasia. An endoscopic ultrasonography may be used to stage esophageal cancer. Capsule endoscopy can show alterations in the esophagus but is more often used for small intestine problems. A barium swallow, capsule endoscopy, and endoscopic ultrasonography cannot provide a definitive diagnosis for cancer.

The patient who is admitted with a diagnosis of diverticulitis and a history of irritable bowel disease and gastroesophageal reflux disease (GERD) has received a dose of Mylanta 30 mL PO. The nurse will determine the medication was effective when which symptom has been resolved? a. Diarrhea b. Heartburn c. Constipation d. Lower abdominal pain

b. Heartburn Mylanta is an antacid that contains both aluminum and magnesium. It is indicated for the relief of gastrointestinal discomfort, such as heartburn associated with GERD. Mylanta can cause both diarrhea and constipation as a side effect. Mylanta does not affect lower abdominal pain.

What information would have the highest priority for the nurse to include in preoperative teaching for a patient scheduled for a colectomy? a. How to care for the wound b. How to deep breathe and cough c. The location and care of drains after surgery d. Which medications will be used during surgery

b. How to deep breathe and cough Because anesthesia, an abdominal incision, and pain can impair the patient's respiratory status in the postoperative period, it is of high priority to teach the patient to cough and deep breathe. Otherwise, the patient could develop atelectasis and pneumonia, which would delay early recovery from surgery and hospital discharge. Care for the wound and location and care of the drains will be briefly discussed preoperatively but will be done again with higher priority after surgery. Knowing which drugs will be used during surgery may not be meaningful to the patient and should be reviewed with the patient by the anesthesiologist.

Priority Decision: Older patients may have cardiac or renal insufficiency and be more susceptible to problems from vomiting and antiemetic drug side effects. What nursing intervention is most important to implement with these patients? a. Keep the patient flat in bed to decrease dizziness. b. hourly visual checks and implement fall precautions. c. Give IV fluids as rapidly as possible to prevent dehydration. d. Keep the patient NPO until nausea and vomiting have stopped.

b. Implementing safety precautions (placement close to the nurses' station, call bell in reach, hourly visual checks) is the priority. The patient would not be kept in a flat position because.fthe potential for aspiration of vomitus. Keeping the patient NPO would be done for all patients but is not the priority with this older patient. Because the older patient is more likely to have cardiac or renal insufficiency, the patient's fluid and electrolyte status are monitored more closely (laboratory, intake and output). Monitor vital signs along with breath sounds. Assess mucous membranes, skin turgor, and color to assess for dehydration. Assess level of consciousness closely. Check dosing of antiemetics. Assess for weakness and fatigue.

Priority Decision: A patient with a gunshot wound to the abdomen reports increasing abdominal pain several hours after surgery to repair the bowel. What action should the nurse take first? a. Notify the HCP. b. Assess the patient's vital signs. c. Position the patient with the knees flexed. d. Determine the patient's IV intake since the end of surgery.

b. It is likely that the patient could be developing peritonitis, which could be life threatening, and assessment of vital signs for hypovolemic shock should be done to report to the HCP. If an IV line is not in place, it should be inserted and pain may be eased by flexing the knees.

A patient was admitted with a fractured hip after being found on the floor of her home. She was extremely malnourished and started on parenteral nutrition (PN) 3 days ago. Which assessment finding would be of most concern to the nurse? a. Blood glucose level of 125 mg/dL b. Serum phosphate level of 1.9 mg/dL c. White blood cell count of 10,500/μL d. Serum potassium level of 4.6 mEq/L

b. Serum phosphate level of 1.9 mg/dL Refeeding syndrome can occur if a malnourished patient is started on aggressive nutritional support. Hypophosphatemia (serum phosphate level <2.4 mg/dL) is the hallmark of refeeding syndrome and could result in cardiac dysrhythmias, respiratory arrest, and neurologic problems. An increase in the blood glucose level is expected during the first few days after PN is started. The goal is to maintain a glucose range of 110 to 150 mg/dL. An elevated white blood cell count (>11,000/μL) could indicate an infection. Normal serum potassium levels are between 3.5 and 5.0 mEq/L.

A patient with ulcerative colitis undergoes the first phase of a total proctocolectomy with ileal pouch and anal anastomosis. On initial postoperative assessment of the patient, what should the nurse expect to find? a. A rectal tube set to low continuous suction b. A loop ileostomy with a plastic rod to hold it in place c. A colostomy stoma with an NG tube in place to provide pouch irrigations d. A permanent ileostomy stoma in the right lower quadrant of the abdomen

b. The initial procedure for a total proctocolectomy with ileal pouch and anal anastomosis includes a colectomy, rectal mucosectomy, ileal reservoir construction, ileoanal anastomosis, and a temporary ileostomy. A loop ileostomy is the most common temporary ileostomy, and it may be held in place with a plastic rod for the first week. A rectal tube to suction is not indicated in any of the surgical procedures for ulcerative colitis. A colostomy is not used, and an NG tube would not be used to irrigate the pouch. A permanent ileostomy stoma would be expected following a total proctocolectomy with a permanent ileostomy.

How is the most common form of malabsorption syndrome treated? a. Administration of antibiotics b. Avoidance of milk and milk products c. Supplementation with pancreatic enzymes d. Avoidance of gluten found in wheat, barley, oats, and rye

b. The most common type of malabsorption syndrome is lactose intolerance, and it is managed by restricting the intake of milk and milk products. Antibiotics are used in cases of bacterial infections that cause malabsorption, pancreatic enzyme supplementation is used for pancreatic insufficiency, and restriction of gluten is necessary for control of adult celiac disease (celiac sprue, glutensensitive enteropathy).

Priority Decision: A patient who has been vomiting for several days from an unknown cause is admitted to the hospital. What should the nurse anticipate will first be included in interprofessional care? a. Oral administration of broth and tea b. IV replacement of fluid and electrolytes c. Administration of parenteral antiemetics d. Insertion of a nasogastric (NG) tube for suction

b. The patient with severe or persistent vomiting requires Ivy replacement of fluids and electrolytes until able to tolerate oral intake to prevent serious dehydration and electrolyte imbalance. Oral fluids are not given until vomiting has been relieved and parenteral anti medics are often not used until a cause of the vomiting can be established. Nasogastric intubation (NG) may be needed in some cases but fluid and electrolyte replacement are the first priority.

The patient has persistent and continuous pain at McBurney's point. The nursing assessment reveals rebound tenderness and muscle guarding with the patient preferring to lie still with the right leg flexed. What should the nursing interventions for this patient include? a. Laxatives to move the constipated bowel b. NPO status in preparation for possible appendectomy c. Parenteral fluids and antibiotic therapy for 6 hours before surgery d. NG tube inserted to decompress the stomach and prevent aspiration

b. The patient's manifestations are characteristic of appendicitis. After laboratory test and CT scan confirmation, the patient will have surgery. Laxatives are not used. The 6 hours of fluids and antibiotics preoperatively would be used if the appendix was ruptured. The NG tube is more likely to be used with abdominal trauma.

A patient who cannot afford enough food for her family states she only eats after her children have eaten. At a clinic visit, she reports bleeding gums; loose teeth; and dry, itchy skin. Which vitamin deficiency would the nurse suspect? a. Folic acid b. Vitamin C c. Vitamin D d. Vitamin K

b. Vitamin C This patient is lacking vitamin C as evidenced by the bleeding gums, loose teeth, and dry, itchy skin. Clinical manifestations of folic acid deficiency include megaloblastic anemia, anorexia, fatigue, sore tongue, diarrhea, or forgetfulness. Clinical manifestations of vitamin D deficiency include muscular weakness, excess sweating, diarrhea, bone pain, rickets, or osteomalacia. Clinical manifestations of vitamin K deficiency include defective blood coagulation.

An older adult patient is seen in the primary care provider's office for a well check complains of difficulty swallowing. What common effect of aging should the nurse assess for as a possible cause? a. Anosmia b. Xerostomia c. Hypochlorhydria d. Salivary gland tumor

b. Xerostomia Xerostomia (decreased saliva production), or dry mouth, affects many older adults and may be associated with difficulty swallowing (dysphagia). Anosmia is loss of sense of smell. Hypochlorhydria, a decrease in stomach acid, does not affect swallowing. Salivary gland tumors are not common.

The patient asks the nurse why she needs to have surgery for a femoral, strangulated hernia. What is the best explanation the nurse can give the patient? a. "The surgery will relieve your constipation." b. "The abnormal hernia must be replaced into the abdomen." c. "The surgery is needed to allow intestinal flow and prevent necrosis." d. "The hernia is because the umbilical opening did not close after birth as it should have."

c. A strangulated femoral hernia obstructs intestinal flow and blood supply, thus requiring emergency surgery. The other options are incorrect.

Which patient would be at highest risk for developing oral candidiasis? a. A 74-yr-old patient who has vitamin B and C deficiencies b. A 22-yr-old patient who smokes 2 packs of cigarettes per day c. A 32-yr-old patient who is receiving ciprofloxacin for 3 weeks d. A 58-yr-old patient who is receiving amphotericin B for 2 days

c. A 32-yr-old patient who is receiving ciprofloxacin for 3 weeks Oral candidiasis is caused by prolonged antibiotic treatment (e.g., ciprofloxacin) or high doses of corticosteroids. Amphotericin B is used to treat candidiasis. Vitamin B and C deficiencies may lead to Vincent's infection. Use of tobacco products leads to stomatitis, not candidiasis.

When caring for the patient with heart failure, the nurse knows that which gastrointestinal process is most dependent on cardiac output and may affect the patient's nutritional status? a. Ingestion b. Digestion c. Absorption d. Elimination

c. Absorption Substances that interface with the absorptive surfaces of the gastrointestinal tract (primarily in the small intestine) diffuse across the intestinal membranes into intestinal capillaries and are then carried to other parts of the body for use in energy production. The cardiac output provides the blood flow for this absorption of nutrients to occur.

The patient with chronic gastritis is being put on a combination of medications to eradicate Helicobacter pylori. Which drugs does the nurse know will probably be used? a. Antibiotic(s), antacid, and corticosteroid b. Antibiotic(s), aspirin, and antiulcer/protectant c. Antibiotic(s), proton pump inhibitor, and bismuth d. Antibiotic(s) and nonsteroidal antiinflammatory drugs (NSAIDs)

c. Antibiotic(s), proton pump inhibitor, and bismuth To eradicate H. pylori, a combination of antibiotics, a proton pump inhibitor, and possibly bismuth (for quadruple therapy) will be used. Corticosteroids, aspirin, and NSAIDs are drugs that can cause gastritis and do not affect H. pylori.

The patient is receiving the following medications. Which one is prescribed to relieve symptoms rather than treat a disease? a. Corticosteroids b. 6-Mercaptopurine c. Antidiarrheal agents d. Sulfasalazine (Azulfidine)

c. Antidiarrheal agents only relieve symptoms. Corticosteroids, 6-mercaptopurine, and sulfasalazine (Azulfidine) are used to treat and control inflammation with various diseases and maintain IBD remission.

Which clinical manifestations of inflammatory bowel disease does the nurse determine are common to both patients with ulcerative colitis (UC) and Crohn's disease? Select all that apply. a Restricted to rectum b. Strictures are common c. Bloody, diarrhea stools d. Cramping abdominal pain e. Lesions penetrate intestine

c. Bloody, diarrhea stools d. Cramping abdominal pain Clinical manifestations of UC and Crohn's disease include bloody diarrhea, cramping abdominal pain, and nutritional disorders. Intestinal lesions associated with UC are usually restricted to the rectum before moving into the colon. Lesions that penetrate the intestine or cause strictures are characteristic of Crohn's disease.

The medications prescribed for the patient with IBD include cobalamin and iron injections. What is the reason for using these drugs? a. Alleviate stress b. Combat infection c. Correct malnutrition d. Improve quality of life

c. Cobalamin and iron injections will help correct malnutrition. Correcting malnutrition will also indirectly help improve quality of life and fight infections.

Corticosteroid medications are associated with the development of peptic ulcers because of which pathophysiologic mechanism? a. The enzyme urease is produced. b. Secretion of hydrochloric acid is increased. c. The rate of mucous cell renewal is decreased. d. The synthesis of mucus and prostaglandins is inhibited.

c. Corticosteroids decrease the rate of mucous cell renewal. H. pylori produces the enzyme urease. Alcohol ingestion increases the secretion of HCI. Aspirin and NSAlDs inhibit the synthesis of mucus and prostaglandins.

A patient who had a gastroduodenostomy (Billroth I operation) for stomach cancer reports generalized weakness, sweating, palpitations, and dizziness 15 to 30 minutes after eating. What long-term complication does the nurse suspect is occurring? a. Malnutrition b. Bile reflux gastritis c. Dumping syndrome d. Postprandial hypoglycemia

c. Dumping syndrome After a Billroth I operation, dumping syndrome may occur 15 to 30 minutes after eating because of the hypertonic fluid going to the intestine and additional fluid being drawn into the bowel. Malnutrition may occur but does not cause these symptoms. Bile reflux gastritis cannot happen when the stomach has been removed. Postprandial hypoglycemia occurs with similar symptoms, but 2 hours after eating.

After an exploratory laparotomy, a patient on a clear liquid diet reports severe gas pains and abdominal distention. Which action by the nurse is most appropriate? a. Return the patient to NPO status. b. Place cool compresses on the abdomen. c. Encourage the patient to ambulate as ordered. d. Administer an as-needed dose of IV morphine sulfate.

c. Encourage the patient to ambulate as ordered. Swallowed air and reduced peristalsis after surgery can result in abdominal distention and gas pains. Early ambulation helps restore peristalsis and eliminate flatus and gas pain. Medications used to reduce gas pain include metoclopramide, which stimulates peristalsis. A heating pad can help to alleviate some of the pain and help make the patient more comfortable. There is no need for the patient to return to NPO status. Drinking ginger ale may be helpful.

The nurse determines a patient has experienced the beneficial effects of therapy with famotidine when which symptom is relieved? a. Nausea b. Belching c. Epigastric pain d. Difficulty swallowing

c. Epigastric pain Famotidine is an H2-receptor antagonist that inhibits parietal cell output of HCl acid and minimizes damage to gastric mucosa related to hyperacidity, thus relieving epigastric pain. It is not indicated for nausea, belching, and dysphagia.

A patient who has dysphagia after a stroke is receiving enteral feedings through a percutaneous endoscopic gastrostomy (PEG). What intervention should the nurse integrate into the plan of care? a. Use 30 mL of normal saline to flush the tube every 4 hours. b. Avoid flushing the tube any time the patient is receiving continuous feedings. c. Flush the tube before and after feedings if the patient's feedings are intermittent. d. Flush the PEG with 100 mL of sterile water before and after medication administration.

c. Flush the tube before and after feedings if the patient's feedings are intermittent. The nurse should flush feeding tubes with 30 mL of water, not normal saline, every 4 hours and before and after medication administration during continuous feeding or before and after intermittent feeding. Flushes of 100 mL are excessive and may cause fluid overload in the patient.

What should the nurse teach the patient with diverticulosis to do? a. Use antibiotics routinely to prevent future inflammation. b. Have an annual colonoscopy to detect malignant changes in the lesions. c. Maintain a high-fiber diet and encourage fluid intake of at least 2 L daily. d. Exclude whole grain breads and cereals from the diet to prevent irritating the bowel.

c. Formation of diverticuli is common when decreased 37. bulk of stool, combined with a more narrowed lumen in the sigmoid colon, causes high intraluminal pressures that result in saccular dilation or outpouching of the mucosa through the muscle of the intestinal wall. To prevent diverticula, fecal volume and passage is increased with use ofhigh-fiber diets and at least 2 L of fluid each day. Bulk laxatives, such as psyllium (Metamucil), may also be used. Antibiotics are used only during acute diverticulitis with infection. Colonoscopies are done to detect problems, but not yearly.

A patient has a sliding hiatal hernia. What nursing intervention will reduce the symptoms of heartburn and dyspepsia? a. Keeping the patient NPO b. Putting the bed in the Trendelenburg position c. Having the patient eat 4 to 6 smaller meals each day d. Giving various antacids to determine which one works for the patient

c. Having the patient eat 4 to 6 smaller meals each day Eating smaller meals during the day will decrease the gastric pressure and symptoms of hiatal hernia. Keeping the patient NPO or in a Trendelenburg position is not safe or realistic for a long period of time for any patient. Varying antacids will only be done with the health care provider's prescription, so this is not a nursing intervention.

The nurse determines that teaching for the patient with peptic ulcer disease has been effective when the patient makes which statement? a. "I should stop all my medications if I develop any side effects." b. "I should continue my treatment regimen as long as I have pain." c. "I have learned some relaxation strategies that decrease my stress." d. "I can buy whatever antacids are on sale because they all have the same effect."

c. Increase vagal stimulation from emotional stress causes hypersecretion of HC1, and stress reduction is an important part of patients management of peptic ulcers, especially duodenal ulcers. If side effects to medications develop, the patient should notify the HCP before altering the drug regimen. Although effective treatment will promote pain relief in several days, the treatment regimen should be continued until there is evidence that the ulcer has healed completely. Interchanging brands and preparations of antacids and histamine receptor blockers without checking the HCPs may cause harmful side effects, and patient should take only prescribed medications.

The patient is having an esophagoenterostomy with anastomosis of a segment of the colon to replace the resected portion. What initial postoperative care should the nurse expect when this patient returns to the nursing unit? a. Turn, deep breathe, cough, and use spirometer every 4 hours. b. Maintain an upright position for at least 2 hours after eating. c. NG will have bloody drainage and it should not be repositioned. d. Keep in a supine position to prevent movement of the anastomosis.

c. NG will have bloody drainage and it should not be repositioned. The patient will have bloody drainage from the nasogastric (NG) tube for 8 to 12 hours, and it should not be repositioned or reinserted without contacting the surgeon. Turning and deep breathing will be done every 2 hours, and the spirometer will be used more often than every 4 hours. Coughing would put too much pressure in the area and should not be done. Because the patient will have the NG tube, the patient will not be eating yet. The patient should be kept in a semi-Fowler's or Fowler's position, not supine, to prevent reflux and aspiration of secretions.

A patient with a history of peptic ulcer disease is hospitalized with symptoms of a perforation. During the initial assessment, what should the nurse expect the patient to report? a. Vomiting of bright-red blood b. Projectile vomiting of undigested food c. Sudden, severe generalized abdominal and back pain d. Hyperactive bowel sounds and upper abdominal swelling

c. Preparation of an ulcer causes sudden, severe abdominal pain that becomes generalized and may be referred to the back, accompanied by a rigid board like abdomen, shallow respirations, and a weak rapid heart rate. Vomiting of blood in the indicates hemmorage of an ulcer. Gastric outlet obstruction is characterized by a projectile vomiting of undigested food, hyperactive stomach sounds, and upper abdominal swelling.

After administration of a dose of metoclopramide, which patient assessment finding would show the medication was effective? a. Decreased blood pressure b. Absence of muscle tremors c. Relief of nausea and vomiting d. No further episodes of diarrhea

c. Relief of nausea and vomiting Metoclopramide is classified as a prokinetic and antiemetic medication. If it is effective, the patient's nausea and vomiting should resolve. Metoclopramide does not affect blood pressure, muscle tremors, or diarrhea.

The nurse is providing care for a patient who is a strict vegetarian. Which dietary choices would the nurse recommend to prevent iron deficiency? a. Brown rice and kidney beans b. Cauliflower and egg substitutes c. Soybeans and hot breakfast cereal d. Whole-grain bread and citrus fruits

c. Soybeans and hot breakfast cereal Vegetarians are at a particular risk for iron deficiency, a problem that can be prevented by regularly consuming high-iron foods such as hot cereals and soybeans. The other foods listed are not classified as high sources of iron.

A patient is diagnosed with celiac disease following a workup for iron-deficiency anemia and decreased bone density. The nurse identifies that additional teaching about disease management is needed when the patient makes which statement? a. "I should ask my close relatives to be screened for celiac disease." b. "If I do not follow the gluten-free diet, I will likely develop malnutrition." c. "I don't need to restrict gluten intake because I don't have diarrhea or bowel symptoms." d. "It is going to be hard to follow a gluten-free diet because it is found in so many foods."

c. The autoimmune process associated with celiac disease continues as long as the body is exposed to gluten, regardless of the symptoms it produces, and a lifelong gluten-free diet is necessary. The other statements regarding celiac disease are all true.

What does the nurse include when teaching a patient with newly diagnosed peptic ulcer disease? a. Maintain a bland, soft, low-residue diet. b. Use alcohol and caffeine in moderation and always with food. c. Eat as normally as possible, eliminating foods that cause pain or discomfort. d. Avoid milk and milk products because they stimulate gastric acid production.

c. There is no specific diet used for the treatment of peptic ulcers, and patients are encouraged to eat as normally as possible, eliminating foods that cause discomfort or pain. Eating 6 meals a day prevents the stomach from being totally empty and is recommended. Caffeine and alcohol should be eliminated from the diet because they are known to cause gastric irritation. Milk and milk products do not have to be avoided but they can add fat content to the diet.

Priority Decision: What is the most important thing the nurse should do when caring for a patient who has contracted Clostridium dificile (C. diff)? a. Clean the entire room with ammonia. b. Feed the patient yogurt with probiotics. c. Wear gloves and wash hands with soap and water. d. Teach the family to use alcohol-based hand cleaners.

c. Wearing gloves will avoid hand contamination. Washing hands with soap and water will remove more Clostridium difficile spores than alcohol-based hand cleaners and ammonia-based disinfectants. The entire room will have to be disinfected with a 10% solution of household bleach. Probiotics may help prevent diarrhea in the patient on antibiotics by replacing normal intestinal bacteria.

A patient was involved in a motor vehicle crash and reports an inability to have a bowel movement. What is the best response by the nurse? a. "You are just too nervous to eat or drink, so there is no stool." b. "Your parasympathetic nervous system is now working to slow the GI tract." c. "The circulation in the GI system has been increased, so less waste is removed." d. "Your sympathetic nervous system was activated, so there is slowing of the GI tract".

d. "Your sympathetic nervous system was activated, so there is slowing of the GI tract". The constipation is most likely related to the sympathetic nervous system activation from the stress related to the accident. Sympathetic nervous system activation can decrease peristalsis. Even without oral intake for a short time, stool will be formed. The parasympathetic system stimulates peristalsis. The circulation to the gastrointestinal system is decreased with stress.

Priority Decision: In instituting a bowel training program for a patient with fecal incontinence, what should the nurse plan to do first? a. Teach the patient to use a perianal pouch. b. Insert a rectal suppository at the same time every morning. c. Place the patient on a bedpan 30 minutes before breakfast. d. Assist the patient to the bathroom at the time of the patient's normal defecation.

d. 2. The first intervention to establish bowel regularity includes promoting bowel evacuation at a regular time each day, preferably by placing the patient on the bedpan, using a bedside commode, or walking the patient to the bathroom. To take advantage of the gastrocolic reflex, an appropriate time is 30 minutes after the first meal of the day or at the patient's usual individual time. Perianal pouches are used to protect the skin only when regularity cannot be established, and evacuation suppositories are also used only if other techniques are not successful.

Priority Decision: When caring for a patient with an acute exacerbation of a peptic ulcer, the nurse finds the patient doubled up in bed with shallow, grunting respirations. Which action should the nurse take first? a. Irrigate the patient's NG tube. b. Notify the health care provider. c. Place the patient in high-Fowler's position. d. Assess the patient's abdomen and vital signs.

d. Abdominal pain that causes the knees to be drawn up and shallow, grunting respirations in a patient with peptic ulcer disease are characterized perforation and the nurse should assess the patients vital signs and abdomen before notifying the HCP. Irrigation of the NG tube should not be done because the added fluid may be spilled into the peritoneal cavity and the patient should be placed in a position of comfort, usually the side with the head slightly elevated.

Following a Billroth Il procedure, a patient develops dumping syndrome. The nurse should explain that the symptoms associated with this problem are caused by? a. distention of the smaller stomach by too much food and fluid intake. b. hyperglycemia caused by uncontrolled gastric emptying into the small intestine. C irritation of the stomach lining by reflux of bile salts because the pylorus has been removed. D movement of fluid into the small bowel from concentrated food and fluids moving rapidly into the intestine.

d. Because there is no sphincter control of food taken into the stomach following a Billroth Il procedure, concentrated food and fluid move rapidly into the small intestine, creating a hypertonic environment that pulls fluid from the bowel wall into the lumen of the intestine, reducing plasma volume and distending the bowel. Postprandial hypoglycemia occurs when the concentrated carbohydrate bolus in the small intestine results in hyperglycemia and the release ofexcessive amounts of insulin into the circulation, resulting in symptoms of hypoglycemia. Irritation of the stomach by bile salts causes epigastric distress after meals, not dumping syndrome.

After identifying that a patient has possible nutritional deficits, which action will the nurse perform next? a. Provide supplements between meals. b. Encourage eating meals with others. c. Have family bring in food from home. d. Complete a full nutritional assessment.

d. Complete a full nutritional assessment. A full nutritional assessment includes history and physical examination and laboratory data. The nutritional assessment will need to be done to provide the basis for nutrition intervention. The interventions may include supplements if ordered, family bringing food from home, and socializing with meals.

The nurse requests a patient scheduled for colectomy to sign the operative permit as directed in the physician's preoperative orders. The patient states that the physician has not really explained very well what is involved in the surgical procedure. What is the most appropriate action by the nurse? a. Ask family members whether they have discussed the surgical procedure with the physician. b. Have the patient sign the form and state the physician will visit to explain the procedure before surgery. c. Explain the planned surgical procedure as well as possible and have the patient sign the consent form. d. Delay the patient's signature on the consent and notify the physician about the conversation with the patient.

d. Delay the patient's signature on the consent and notify the physician about the conversation with the patient. The patient should not be asked to sign a consent form unless the procedure has been explained to the satisfaction of the patient. The nurse should notify the physician, who has the responsibility for obtaining consent.

The nurse is caring for a patient admitted with a suspected bowel obstruction. The nurse auscultating the abdomen listens for which type of bowel sounds that are consistent with the patient's clinical picture? a. Low-pitched and rumbling above the area of obstruction b. High-pitched and hypoactive below the area of obstruction c. Low-pitched and hyperactive below the area of obstruction d. High-pitched and hyperactive above the area of obstruction

d. High-pitched and hyperactive above the area of obstruction Early in intestinal obstruction, the patient's bowel sounds are hyperactive and high pitched, sometimes referred to as "tinkling," above the level of the obstruction. This occurs because peristaltic action increases to "push past" the area of obstruction. As the obstruction becomes complete, bowel sounds decrease and finally become absent.

A patient with upper GI bleeding and melena is treated with several drugs. Which drug should the nurse recognize as a priority to administer before, during, and potentially after endoscopy? a. Oral nizatidine (Axid) b. Epinephrine injection c. Vasopressin injection esomeprazole (Nexium)

d. IV esomeprazole (Nexium) is a proton pump inhibitor (PPI) that is used to decrease acid secretion and prevent interference with clotting as a bolus before endoscopy and then a continuous infusion. Nizatidine is a histamine (HD-receptor blocker that decreases acid secretion but is not as effective as PPIs. Epinephrine injection during endoscopy is effective for acute hemostasis. Vasopressin has a vasoconstriction action useful in controlling upper GI bleeding but does not facilitate clotting.

Priority Decision: While caring for a patient following a subtotal gastrectomy with a gastroduodenostomy anastomosis, the nurse determines that the NG tube is obstructed. Which action should the nurse take first? a. Replace the tube with a new one. b. Irrigate the tube until return can be aspirated. c. Reposition the tube and then attempt irrigation. d. Notify the surgeon to reposition or replace the tube.

d. If the patient's NG tube becomes obstructed following a gastrectomy with an intestinal anastomosis, gastric secretions may put a strain on the sutured anastomosis and cause serious complications. Be sure that the suction is working and because of the danger of perforating the gastric mucosa or disrupting the suture line, the nurse should notify the surgeon. Periodic gentle irrigation with normal saline solution may be ordered or the surgeon may choose to reposition or replace the NG tube.

The patient comes to the ED with intermittent crampy abdominal pain, nausea, projectile vomiting, and dehydration. The nurse suspects a GI obstruction. Based on the manifestations, what area of the bowel should the nurse suspect is obstructed? a. Large intestine b. Esophageal sphincter c. Distal small intestine d. Proximal small intestine

d. Intermittent crampy abdominal pain, nausea, projectile vomiting, and dehydration are characteristics of proximal small intestinal obstruction. Large bowel obstruction is characterized by constipation, low-grade abdominal pain, and abdominal distention. Esophageal sphincter blockage or achalasia feels like food is stuck in the chest. Fecal emesis is seen with distal small intestinal obstruction.

What is the rationale for treating acute exacerbation of peptic ulcer disease with NG intubation? a. Stop spillage of GI contents into the peritoneal cavity. b. Remove excess fluids and undigested food from the stomach. c. Feed the patient the nutrients missing from the lack of ingestion. d. Remove stimulation for hydrochloric acid (HCI) acid and pepsin secretion by keeping the stomach empty.

d. NG intubation is used with acute exacerbation of peptic ulcer disease (PUD) to remove the stimulation for HCI and pepsin secretion by keeping the stomach empty. Stopping the spillage of GI contents into the peritoneal cavity is used for peritonitis. Removing excess fluids and undigested food from the stomach is the rationale for using NG intubation for gastric outlet obstruction.

A patient was admitted with epigastric pain because of a gastric ulcer. Which patient assessment warrants an urgent change in the nursing plan of care? a. Back pain 3 or 4 hours after eating a meal b. Chest pain relieved with eating or drinking water c. Burning epigastric pain 90 minutes after breakfast d. Rigid abdomen and vomiting following indigestion

d. Rigid abdomen and vomiting following indigestion A rigid abdomen with vomiting in a patient who has a gastric ulcer indicates a perforation of the ulcer, especially if the manifestations of perforation appear suddenly. Midepigastric pain is relieved by eating, drinking water, or antacids with duodenal ulcers, not gastric ulcers. Back pain 3 to 4 hours after a meal is more likely to occur with a duodenal ulcer. Burning epigastric pain 1 to 2 hours after a meal is an expected manifestation of a gastric ulcer related to increased gastric secretions and does not cause an urgent change in the nursing plan of care.

A male patient who is scheduled for an abdominal-perineal resection (APR) is worried about his sexuality. What is the best nursing intervention for this patient? a. Have the patient's sexual partner reassure the patient that he is still desirable. b. Reassure the patient that sexual function will return when healing is complete. c. Remind the patient that affection can be expressed in ways other than through sexual intercourse. d. Explain that physical and emotional factors can affect sexual function but not necessarily the patient's sexuality.

d. Sexual dysfunction may result from an APR, but the nurse should discuss with the patient that different nerve pathways affect erection, ejaculation, and orgasm and that a dysfunction of one does not mean total sexual dysfunction and also that an alteration in sexual activity does not have to alter sexuality. Referral to a wound, ostomy, and continence nurse (WOCN) would also be helpful. Simple reassurance of desirability and ignoring concerns about sexual function do not help the patient regain positive feelings of sexuality.

The patient with a new ileostomy needs discharge teaching. What should the nurse plan to include in this teaching? a. The pouch can be worn for up to 2 weeks before changing it. b. Decrease the amount of fluid intake to decrease the amount of drainage. c. The pouch can be removed when bowel movements have been regulated. d. If leakage occurs, promptly remove the pouch, clean the skin, and apply a new pouch.

d. The ileostomy drainage is extremely irritating to the skin, so the skin must be cleaned and a new solid skin barrier and pouch applied as soon as a leak occurs to prevent skin damage. The pouch is usually worn for 4 to 7 days unless there is a leak. Because the initial drainage from the ileostomy is high, the fluid intake must be increased. The pouch must always be worn, as the liquid drainage, not formed bowel movements, is frequent.b.

What physiologically occurs with vomiting? a. The acid—base imbalance most commonly associated with persistent vomiting is metabolic acidosis caused by loss of bicarbonate. b. Stimulation of the vomiting center by the chemoreceptor trigger zone (CTZ) is commonly caused by stretch and distension of hollow organs. c. Vomiting requires the coordination of activities of structures including the glottis, respiratory expiration, relaxation of the pylorus, and closure of the lower esophageal sphincter (LES) d. Immediately before the act of vomiting, activation of the parasympathetic nervous system causes increased salivation, increased gastric motility, and relaxation of the LES.

d. The parasympathetic nervous system causes increased may occur with prolonged NPO status and results in salivation and gastric mobility as well as relaxation decreased saliva and ear pain. Oral candidiasis is seen of the lower esophageal sphincter. The acid-base imbalance that occurs with vomiting is metabolic alkalosis from the loss of hydrochloric acid (HCI). The vomiting center in the chemoreceptor trigger zone (CTZ) can be caused by chemical stimuli of drugs, toxins, and labyrinthine stimulation. Vomiting requires the coordination of closing the glottis, deep inspiration with contraction of the abdominal muscles

A 22-year-old patient calls the outpatient clinic reporting nausea and vomiting and right lower abdominal pain. What should the nurse advise the patient to do? a. Use a heating pad to relax the muscles at the site of the pain. b. Drink at least 2 quarts of juice to replace the fluid lost in vomiting. c. Take a laxative to empty the bowel before examination at the clinic. d. Have the symptoms evaluated right away by a health care provider (HCP) at a hospital's ED.

d. The patient is having symptoms of an acute abdomen and should be evaluated immediately by an HCP at a hospital able to perform surgery if needed. The patient's age, location of pain, and other symptoms are characteristic of appendicitis. Heat application and laxatives should not be used in patients with undiagnosed abdominal pain because they may cause perforation of the appendix or other inflammations. Fluids should not be taken until vomiting is controlled, nor should they be taken in the event that surgery may be performed.

In report, the nurse learns that the patient has a transverse colostomy. What should the nurse expect when providing care for this patient? a. Semiliquid stools with increased fluid requirements b. Liquid stools in a pouch and increased fluid requirements c. Formed stools with a pouch, needing irrigation, but no fluid needs d. Semiformed stools in a pouch with the need to monitor fluid balance

d. The patient with a transverse colostomy has semiliquid to semiformed stools needing a pouch and needs to have fluid balance monitored. The ascending colostomy has semiliquid stools needing a pouch and increased fluid. The ileostomy has liquid to semiliquid stools needing a pouch and increased fluid. The sigmoid colostomy has formed stools, may or may not need a pouch but will need irrigation, and no changes in fluid needs.

The nurse evaluates that management of the patient with upper GI bleeding is effective when assessment and laboratory findings reveal which result? a. Hematocrit (Hct) of 35% b. Urinary output of 20 mL/hr c. Urine specific gravity of 1.030 d. Decreasing blood urea nitrogen (BUN)

d. The patient's blood urea nitrogen (BUN) is usually elevated with a significant hemorrhage because blood proteins are subject to bacteria or breakdown in the GI tract. With control of bleeding, the BUN will return to normal period during the early stage of bleeding, that hematocrit is not always a reliable indicator of the amount of blood lost or the amount of blood replaced and may be falsely high or low. A urine output of less than or equal to 20 ml/hr indicates impaired renal perfusion and hypovolemia and a urine specific gravity of 1.030 indicates concentrated urine typical of hypovolemia.

What extraintestinal manifestations are seen in both ulcerative colitis and Crohn's disease? a. Celiac disease and gallstones b. Peptic ulcer disease and uveitis c. Conjunctivitis and colonic dilation d. Erythema nodosum and osteoporosis

d. Ulcerative colitis and Crohn's disease have many of the same extraintestinal symptoms, including erythema nodosum and osteoporosis, as well as gallstones, uveitis, and conjunctivitis. Celiac disease, peptic ulcer disease, and colonic dilation are not extraintestinal.

A patient received a small-bore nasogastric (NG) tube after a laryngectomy. Which action has the highest priority before initiating enteral feedings? a. Testing aspirated fluid pH b. Auscultating while instilling air c. Elevating head of bed to 40 degrees d. Verifying NG tube placement on x-ray

d. Verifying NG tube placement on x-ray It is imperative to ensure that an NG tube is in the gastrointestinal tract rather than the patient's lungs. When an NG tube has been recently inserted, it is important to confirm this placement with an x-ray that will identify the tube's radiopaque tip. Aspiration and air auscultation may not differentiate between gastric and respiratory placement of the tube. Although elevating the head of bed at least 30 degrees is necessary to prevent aspiration, placement must first be confirmed before initiating feedings.

When a patient returns to the clinical unit after an abdominal-perineal resection (APR), what should the nurse expect the patient to have? a. An abdominal dressing b. An abdominal wound and drains c. A temporary colostomy and drains d. A perineal wound, drains, and a stoma

d. With an abdominal perineal-resection (APR), an abdominal incision is made, and the proximal sigmoid colon is brought through the abdominal wall and formed into a permanent colostomy. The patient is repositioned, a perineal incision is made, and the distal sigmoid colon, rectum, and anus are removed through the perineal incision, which may be closed or open and packed, and have drains.


Kaugnay na mga set ng pag-aaral

State laws , rules, and regulations

View Set

Chapter 4: Types of Life Policies

View Set

bio 102 semester 1 midterm! plants! keith!

View Set

3 Strengths and Weaknesses Questions

View Set

Pharmacology Chapter 36 - Multiple Choice

View Set

pn fundamentals online practice 2020 B

View Set